Download as pdf or txt
Download as pdf or txt
You are on page 1of 47

Unit 6

Equilibrium

Chemical equilibria are important in numerous biological


and environmental processes. For example, equilibria
involving O2 molecules and the protein hemoglobin play
After studying this unit you will be
a crucial role in the transport and delivery of O2 from
able to
our lungs to our muscles. Similar equilibria involving CO
• identify dynamic nature of molecules and hemoglobin account for the toxicity of CO.
equilibrium involved in physical
and chemical processes; When a liquid evaporates in a closed container,
• state the law of equilibrium; molecules with relatively higher kinetic energy escape
• explain characteristics of the liquid surface into the vapour phase and number of
equilibria involved in physical liquid molecules from the vapour phase strike the liquid
and chemical processes; surface and are retained in the liquid phase. It gives rise
• write expressions for equilibrium to a constant vapour pressure because of an equilibrium in
constants;
which the number of molecules leaving the liquid equals the
• establish a relationship between
Kp and Kc; number returning to liquid from the vapour. We say that
• explain various factors that the system has reached equilibrium state at this stage.
affect the equilibrium state of a However, this is not static equilibrium and there is a lot of
reaction; activity at the boundary between the liquid and the vapour.
• classify substances as acids or Thus, at equilibrium, the rate of evaporation is equal to the
bases according to Arrhenius, rate of condensation. It may be represented by
Bronsted-Lowry and Lewis
concepts; H2O (l) H2O (vap)
• classify acids and bases as
The double half arrows indicate that the processes
weak or strong in terms of their
ionization constants; in both the directions are going on simultaneously. The
• explain the dependence of degree mixture of reactants and products in the equilibrium state
of ionization on concentration is called an equilibrium mixture.
of the electrolyte and that of the
common ion; Equilibrium can be established for both physical
• describe pH scale for representing processes and chemical reactions. The reaction may be
hydrogen ion concentration; fast or slow depending on the experimental conditions and
• explain ionisation of water and the nature of the reactants. When the reactants in a closed
its duel role as acid and base; vessel at a particular temperature react to give products,
• describe ionic product (Kw ) and the concentrations of the reactants keep on decreasing,
pKw for water;
while those of products keep on increasing for some time
• appreciate use of buffer
after which there is no change in the concentrations
solutions;
• calculate solubility product
of either of the reactants or products. This stage of the
constant. system is the dynamic equilibrium and the rates of the
forward and reverse reactions become equal. It is due to

Rationalised 2023-24

Unit 6.indd 168 9/12/2022 11:58:20 AM


EQUILIBRIUM 169

this dynamic equilibrium stage that there is characteristic features. We observe that the
no change in the concentrations of various mass of ice and water do not change with
species in the reaction mixture. Based on the time and the temperature remains constant.
extent to which the reactions proceed to reach However, the equilibrium is not static.
the state of chemical equilibrium, these may The intense activity can be noticed at the
be classified in three groups. boundary between ice and water. Molecules
(i) The reactions that proceed nearly from the liquid water collide against ice and
to completion and only negligible adhere to it and some molecules of ice escape
concentrations of the reactants are into liquid phase. There is no change of mass
left. In some cases, it may not be even of ice and water, as the rates of transfer of
possible to detect these experimentally. molecules from ice into water and of reverse
transfer from water into ice are equal at
(ii) The reactions in which only small
atmospheric pressure and 273 K.
amounts of products are formed and
most of the reactants remain unchanged It is obvious that ice and water are in
at equilibrium stage. equilibrium only at particular temperature
and pressure. For any pure substance at
(iii) The reactions in which the concentrations
atmospheric pressure, the temperature at
of the reactants and products are
which the solid and liquid phases are at
comparable, when the system is in
equilibrium is called the normal melting point
equilibrium.
or normal freezing point of the substance. The
The extent of a reaction in equilibrium system here is in dynamic equilibrium and we
varies with the experimental conditions such can infer the following:
as concentrations of reactants, temperature, (i) Both the opposing processes occur
etc. Optimisation of the operational conditions simultaneously.
is very important in industry and laboratory (ii) Both the processes occur at the same
so that equilibrium is favorable in the rate so that the amount of ice and water
direction of the desired product. Some remains constant.
important aspects of equilibrium involving
physical and chemical processes are dealt in 6.1.2 Liquid-Vapour Equilibrium
this unit along with the equilibrium involving This equilibrium can be better understood if
ions in aqueous solutions which is called as we consider the example of a transparent box
ionic equilibrium. carrying a U-tube with mercury (manometer).
Drying agent like anhydrous calcium chloride
6.1 E Q U IL IBRI U M IN P H Y S I C A L
(or phosphorus penta-oxide) is placed for
PROCESSES
a few hours in the box. After removing the
The characteristics of system at equilibrium drying agent by tilting the box on one side, a
are better understood if we examine some watch glass (or petri dish) containing water
physical processes. The most familiar examples is quickly placed inside the box. It will be
are phase transformation processes, e.g., observed that the mercury level in the right
solid liquid limb of the manometer slowly increases and
liquid gas finally attains a constant value, that is, the
solid gas pressure inside the box increases and reaches
a constant value. Also the volume of water in
6.1.1 Solid-Liquid Equilibrium
the watch glass decreases (Fig. 6.1). Initially
Ice and water kept in a perfectly insulated there was no water vapour (or very less) inside
thermos flask (no exchange of heat between its the box. As water evaporated the pressure in
contents and the surroundings) at 273K and the box increased due to addition of water
the atmospheric pressure are in equilibrium molecules into the gaseous phase inside
state and the system shows interesting the box. The rate of evaporation is constant.

Rationalised 2023-24

Unit 6.indd 169 9/12/2022 11:58:20 AM


170 chemistry

Fig. 6.1 Measuring equilibrium vapour pressure of water at a constant temperature

However, the rate of increase in pressure vapour to liquid state is much less than the
decreases with time due to condensation rate of evaporation. These are open systems
of vapour into water. Finally it leads to an and it is not possible to reach equilibrium in
equilibrium condition when there is no net an open system.
evaporation. This implies that the number
Water and water vapour are in equilibrium
of water molecules from the gaseous state
into the liquid state also increases till the position at atmospheric pressure (1.013 bar)
equilibrium is attained i.e., and at 100°C in a closed vessel. The boiling
point of water is 100°C at 1.013 bar pressure.
rate of evaporation= rate of condensation
For any pure liquid at one atmospheric
H2O(l) H2O (vap)
pressure (1.013 bar), the temperature
At equilibrium the pressure exerted by at which the liquid and vapours are at
the water molecules at a given temperature equilibrium is called normal boiling point of
remains constant and is called the equilibrium the liquid. Boiling point of the liquid depends
vapour pressure of water (or just vapour on the atmospheric pressure. It depends on
pressure of water); vapour pressure of water the altitude of the place; at high altitude the
increases with temperature. If the above boiling point decreases.
experiment is repeated with methyl alcohol,
acetone and ether, it is observed that different 6.1.3 Solid – Vapour Equilibrium
liquids have different equilibrium vapour
Let us now consider the systems where solids
pressures at the same temperature, and the
sublime to vapour phase. If we place solid
liquid which has a higher vapour pressure is
iodine in a closed vessel, after sometime
more volatile and has a lower boiling point.
the vessel gets filled up with violet vapour
If we expose three watch glasses containing and the intensity of colour increases with
separately 1mL each of acetone, ethyl alcohol, time. After certain time the intensity of
and water to atmosphere and repeat the colour becomes constant and at this stage
experiment with different volumes of the
equilibrium is attained. Hence solid iodine
liquids in a warmer room, it is observed
sublimes to give iodine vapour and the iodine
that in all such cases the liquid eventually
vapour condenses to give solid iodine. The
disappears and the time taken for complete
equilibrium can be represented as,
evaporation depends on (i) the nature of the
liquid, (ii) the amount of the liquid and (iii) the I2(solid) I2 (vapour)
temperature. When the watch glass is open Other examples showing this kind of
to the atmosphere, the rate of evaporation equilibrium are,
remains constant but the molecules are
dispersed into large volume of the room. As Camphor (solid) Camphor (vapour)
a consequence the rate of condensation from NH4Cl (solid) NH4Cl (vapour)

Rationalised 2023-24

Unit 6.indd 170 9/12/2022 11:58:21 AM


EQUILIBRIUM 171

6.1.4 Equilibrium Involving Dissolution pressure of the gas above the solvent.
of Solid or Gases in Liquids This amount decreases with increase of
Solids in liquids temperature. The soda water bottle is sealed
under pressure of gas when its solubility in
We know from our experience that we can
water is high. As soon as the bottle is opened,
dissolve only a limited amount of salt or
some of the dissolved carbon dioxide gas
sugar in a given amount of water at room
escapes to reach a new equilibrium condition
temperature. If we make a thick sugar syrup
required for the lower pressure, namely its
solution by dissolving sugar at a higher
partial pressure in the atmosphere. This is
temperature, sugar crystals separate out if we
how the soda water in bottle when left open
cool the syrup to the room temperature. We call
to the air for some time, turns ‘flat’. It can be
it a saturated solution when no more of solute
generalised that:
can be dissolved in it at a given temperature.
The concentration of the solute in a saturated (i) For solid liquid equilibrium, there is
solution depends upon the temperature. In only one temperature (melting point) at
a saturated solution, a dynamic equilibrium 1 atm (1.013 bar) at which the two
exits between the solute molecules in the solid phases can coexist. If there is no
state and in the solution: exchange of heat with the surroundings,
the mass of the two phases remains
Sugar (solution) Sugar (solid), and
constant.
the rate of dissolution of sugar = rate of
crystallisation of sugar. (ii) For liquid vapour equilibrium, the
vapour pressure is constant at a given
Equality of the two rates and dynamic
temperature.
nature of equilibrium has been confirmed with
the help of radioactive sugar. If we drop some (iii) For dissolution of solids in liquids,
radioactive sugar into saturated solution of the solubility is constant at a given
non-radioactive sugar, then after some time temperature.
radioactivity is observed both in the solution (iv) For dissolution of gases in liquids,
and in the solid sugar. Initially there were no the concentration of a gas in liquid
radioactive sugar molecules in the solution is proportional to the pressure
but due to dynamic nature of equilibrium, (concentration) of the gas over the liquid.
there is exchange between the radioactive These observations are summarised in
and non-radioactive sugar molecules between Table 6.1
the two phases. The ratio of the radioactive Table 6.1 Some Features of Physical Equilibria
to non-radioactive molecules in the solution
increases till it attains a constant value. Process Conclusion
Liquid Vapour p H 2O constant at given
Gases in liquids temperature
H2O (l) H2O (g)
When a soda water bottle is opened, some of
Solid Liquid Melting point is fixed at
the carbon dioxide gas dissolved in it fizzes constant pressure
out rapidly. The phenomenon arises due H2O (s) H2O (l)
to difference in solubility of carbon dioxide Solute(s) Solute Concentration of solute
at different pressures. There is equilibrium (solution) in solution is constant
between the molecules in the gaseous state Sugar(s) Sugar at a given temperature
and the molecules dissolved in the liquid (solution)
under pressure i.e.,
Gas(g) Gas (aq) [gas(aq)]/[gas(g)] is
CO2 (gas) CO2 (in solution) constant at a given
This equilibrium is governed by Henry’s temperature
CO2(g) CO2(aq)
law, which states that the mass of a gas [CO2(aq)]/[CO2(g)] is
dissolved in a given mass of a solvent at constant at a given
any temperature is proportional to the temperature

Rationalised 2023-24

Unit 6.indd 171 11/2/2022 4:10:42 PM


172 chemistry

6.1.5 General Characteristics of


Equilibria Involving Physical
Processes
For the physical processes discussed above,
following characteristics are common to the
system at equilibrium:
(i) Equilibrium is possible only in a closed
system at a given temperature.
(ii) Both the opposing processes occur at
the same rate and there is a dynamic
but stable condition.
(iii) All measurable properties of the system
remain constant.
(iv) When equilibrium is attained for a
physical process, it is characterised by Fig. 6.2 Attainment of chemical equilibrium.
constant value of one of its parameters
at a given temperature. Table 6.1 lists Eventually, the two reactions occur at the
such quantities. same rate and the system reaches a state of
(v) The magnitude of such quantities at any equilibrium.
stage indicates the extent to which the Similarly, the reaction can reach the state
physical process has proceeded before of equilibrium even if we start with only C and
reaching equilibrium. D; that is, no A and B being present initially,
as the equilibrium can be reached from either
6.2 EQUILIBRIUM IN CHEMICAL direction.
PROCESSES – DYNAMIC
EQUILIBRIUM The dynamic nature of chemical
equilibrium can be demonstrated in the
Analogous to the physical systems chemical synthesis of ammonia by Haber’s process.
reactions also attain a state of equilibrium. In a series of experiments, Haber started
These reactions can occur both in forward with known amounts of dinitrogen and
and backward directions. When the rates of dihydrogen maintained at high temperature
the forward and reverse reactions become and pressure and at regular intervals
equal, the concentrations of the reactants determined the amount of ammonia present.
and the products remain constant. This He was successful in determining also the
is the stage of chemical equilibrium. This concentration of unreacted dihydrogen and
equilibrium is dynamic in nature as it consists dinitrogen. Fig. 6.4 (page 174) shows that after
of a forward reaction in which the reactants a certain time the composition of the mixture
give product(s) and reverse reaction in which remains the same even though some of the
product(s) gives the original reactants. reactants are still present. This constancy in
For a better comprehension, let us consider composition indicates that the reaction has
a general case of a reversible reaction, reached equilibrium. In order to understand
A+B C+D the dynamic nature of the reaction, synthesis
of ammonia is carried out with exactly the
With passage of time, there is same starting conditions (of partial pressure
accumulation of the products C and D and and temperature) but using D2 (deuterium)
depletion of the reactants A and B (Fig. 6.2). in place of H2. The reaction mixtures starting
This leads to a decrease in the rate of either with H2 or D2 reach equilibrium with
forward reaction and an increase in the rate the same composition, except that D2 and
of the reverse reaction, ND3 are present instead of H2 and NH3. After

Rationalised 2023-24

Unit 6.indd 172 11/2/2022 4:10:43 PM


EQUILIBRIUM 173

Dynamic Equilibrium – A Student’s Activity


Equilibrium whether in a physical or in a chemical system, is always of dynamic nature. This can
be demonstrated by the use of radioactive isotopes. This is not feasible in a school laboratory.
However this concept can be easily comprehended by performing the following activity. The activity
can be performed in a group of 5 or 6 students.
Take two 100mL measuring cylinders (marked as 1 and 2) and two glass tubes each of
30 cm length. Diameter of the tubes may be same or different in the range of 3-5mm. Fill
nearly half of the measuring cylinder-1 with coloured water (for this purpose add a crystal
of potassium permanganate to water) and keep second cylinder (number 2) empty.
Put one tube in cylinder 1 and second in cylinder 2. Immerse one tube in cylinder 1,
close its upper tip with a finger and transfer the coloured water contained in its lower portion to
cylinder 2. Using second tube, kept in 2nd cylinder, transfer the coloured water in a similar manner
from cylinder 2 to cylinder 1. In this way keep on transferring coloured water using the two glass
tubes from cylinder 1 to 2 and from 2 to 1 till you notice that the level of coloured water in both
the cylinders becomes constant.
If you continue intertransferring coloured solution between the cylinders, there will not be
any further change in the levels of coloured water in two cylinders. If we take analogy of ‘level’ of
coloured water with ‘concentration’ of reactants and products in the two cylinders, we can say
the process of transfer, which continues even after the constancy of level, is indicative of dynamic
nature of the process. If we repeat the experiment taking two tubes of different diameters we find
that at equilibrium the level of coloured water in two cylinders is different. How far diameters are
responsible for change in levels in two cylinders? Empty cylinder (2) is an indicator of no product
in it at the beginning.

1 2 1 2

(a) (b)
Fig.6.3 Demonstrating dynamic nature of equilibrium. (a) initial stage (b) final stage after the
equilibrium is attained.

Rationalised 2023-24

Unit 6.indd 173 9/12/2022 11:58:21 AM


174 chemistry

2NH3(g) N2(g) + 3H2(g)


Similarly let us consider the reaction,
H 2 (g) + I 2 (g) 2HI(g). If we start with
equal initial concentration of H2 and I2, the
reaction proceeds in the forward direction
and the concentration of H2 and I2 decreases
while that of HI increases, until all of these
become constant at equilibrium (Fig. 6.5). We
can also start with HI alone and make the
reaction to proceed in the reverse direction;
the concentration of HI will decrease and
concentration of H2 and I2 will increase until
they all become constant when equilibrium is
reached (Fig. 6.5). If total number of H and I
atoms are same in a given volume, the same
equilibrium mixture is obtained whether we
Fig. 6.4 Depiction of equilibrium for the reaction
start it from pure reactants or pure product.
N 2  g   3H 2  g  2NH3  g 

equilibrium is attained, these two mixtures


(H 2, N 2, NH 3 and D 2, N 2, ND 3) are mixed
together and left for a while. Later, when
this mixture is analysed, it is found that the
concentration of ammonia is just the same
as before. However, when this mixture is
analysed by a mass spectrometer, it is found
that ammonia and all deuterium containing
forms of ammonia (NH3, NH2D, NHD2 and
ND3) and dihydrogen and its deutrated forms
(H2, HD and D2) are present. Thus one can
conclude that scrambling of H and D atoms in
the molecules must result from a continuation
of the forward and reverse reactions in the
mixture. If the reaction had simply stopped
Fig. 6.5 Chemical equilibrium in the reaction H2(g)
when they reached equilibrium, then there
+ I2(g) 2HI(g) can be attained from
would have been no mixing of isotopes in either direction
this way.
6.3 LAW OF CHEMICAL EQUILIBRIUM
Use of isotope (deuterium) in the formation AND EQUILIBRIUM CONSTANT
of ammonia clearly indicates that chemical
A mixture of reactants and products in the
reactions reach a state of dynamic
equilibrium state is called an equilibrium
equilibrium in which the rates of forward
mixture. In this section we shall address a
and reverse reactions are equal and there
number of important questions about the
is no net change in composition.
composition of equilibrium mixtures: What is
Equilibrium can be attained from both the relationship between the concentrations
sides, whether we start reaction by taking, of reactants and products in an equilibrium
H2(g) and N2(g) and get NH3(g) or by taking mixture? How can we determine equilibrium
NH3(g) and decomposing it into N2(g) and H2(g). concentrations from initial concentrations?
N2(g) + 3H2(g) 2NH3(g) What factors can be exploited to alter the

Rationalised 2023-24

Unit 6.indd 174 9/12/2022 11:58:21 AM


EQUILIBRIUM 175

composition of an equilibrium mixture? The Six sets of experiments with varying initial
last question in particular is important when conditions were performed, starting with only
choosing conditions for synthesis of industrial gaseous H2 and I2 in a sealed reaction vessel
chemicals such as H2, NH3, CaO etc. in first four experiments (1, 2, 3 and 4) and
To answer these questions, let us consider only HI in other two experiments (5 and 6).
a general reversible reaction: Experiment 1, 2, 3 and 4 were performed
A+B C+D taking different concentrations of H2 and / or
where A and B are the reactants, C and D I2, and with time it was observed that intensity
are the products in the balanced chemical of the purple colour remained constant and
equation. On the basis of experimental studies equilibrium was attained. Similarly, for
of many reversible reactions, the Norwegian experiments 5 and 6, the equilibrium was
chemists Cato Maximillian Guldberg and attained from the opposite direction.
Peter Waage proposed in 1864 that the Data obtained from all six sets of
concentrations in an equilibrium mixture experiments are given in Table 6.2.
are related by the following equilibrium
equation, It is evident from the experiments 1, 2,
3 and 4 that number of moles of dihydrogen
Kc 
CD         (6.1) reacted = number of moles of iodine reacted
 A  B  = ½ (number of moles of HI formed). Also,
(6.1) where Kc is the equilibrium constant and experiments 5 and 6 indicate that,
the expression on the right side is called the [H2(g)]eq = [I2(g)]eq
equilibrium constant expression.
The equilibrium equation is also known as Knowing the above facts, in order
the law of mass action because in the early to establish a relationship between
days of chemistry, concentration was called concentrations of the reactants and products,
“active mass”. In order to appreciate their several combinations can be tried. Let us
work better, let us consider reaction between consider the simple expression,
gaseous H2 and I2 carried out in a sealed vessel [HI(g)]eq / [H2(g)]eq [I2(g)]eq
at 731K.
H2(g) + I2(g)   2HI(g) It can be seen from Table 6.3 that if we
1 mol 1 mol 2 mol put the equilibrium concentrations of the
reactants and products, the above expression

Table 6.2 Initial and Equilibrium Concentrations of H2, I2 and HI

Rationalised 2023-24

Unit 6.indd 175 9/12/2022 11:58:21 AM


176 chemistry

Table 6.3 Expression Involving the The equilibrium constant for a general
Equilibrium Concentration of reaction,
Reactants
aA + bB cC + dD
H2(g) + I2(g) 2HI(g)
is expressed as,
Kc = [C]c[D]d / [A]a[B]b (6.4)

where [A], [B], [C] and [D] are the equilibrium


concentrations of the reactants and products.
Equilibrium constant for the reaction,
4NH3(g) + 5O2(g) 4NO(g) + 6H2O(g) is written
as
Kc = [NO]4[H2O]6 / [NH3]4 [O2]5
Molar concentration of different species is
is far from constant. However, if we consider indicated by enclosing these in square bracket
the expression, and, as mentioned above, it is implied that
[HI(g)]2eq / [H2(g)]eq [I2(g)]eq these are equilibrium concentrations. While
writing expression for equilibrium constant,
we find that this expression gives constant
symbol for phases (s, l, g) are generally ignored.
value (as shown in Table 6.3) in all the six
cases. It can be seen that in this expression Let us write equilibrium constant for the
the power of the concentration for reactants reaction, H2(g) + I2(g) 2HI(g) (6.5)
and products are actually the stoichiometric
as, Kc = [HI]2 / [H2] [I2] = x (6.6)
coefficients in the equation for the chemical
reaction. Thus, for the reaction H2(g) + I2(g) The equilibrium constant for the reverse
2HI(g), following equation 6.1, the equilibrium reaction, 2HI(g) H2(g) + I2(g), at the same
constant Kc is written as, temperature is,
Kc = [HI(g)]eq2 / [H2(g)]eq [I2(g)]eq (6.2) K′c = [H2] [I2] / [HI]2 = 1/ x = 1 / Kc (6.7)
Generally the subscript ‘eq’ (used for Thus, K′c = 1 / Kc (6.8)
equilibrium) is omitted from the concentration Equilibrium constant for the reverse
terms. It is taken for granted that the reaction is the inverse of the equilibrium
concentrations in the expression for Kc are constant for the reaction in the forward
equilibrium values. We, therefore, write, direction.
Kc = [HI(g)]2 / [H2(g)] [I2(g)] (6.3) If we change the stoichiometric coefficients
in a chemical equation by multiplying
The subscript ‘c’ indicates that K c is
throughout by a factor then we must make
expressed in concentrations of mol L–1.
sure that the expression for equilibrium
At a given temperature, the product of constant also reflects that change. For
concentrations of the reaction products example, if the reaction (6.5) is written as,
raised to the respective stoichiometric
coefficient in the balanced chemical ½ H2(g) + ½ I2(g) HI(g) (6.9)
equation divided by the product of the equilibrium constant for the above
concentrations of the reactants raised to reaction is given by
their individual stoichiometric coefficients K″c = [HI] / [H2]1/2[I2]1/2 = {[HI]2 / [H2][I2]}1/2
has a constant value. This is known as
the Equilibrium Law or Law of Chemical = x1/2 = Kc1/2 (6.10)
Equilibrium. On multiplying the equation (6.5) by n, we get

Rationalised 2023-24

Unit 6.indd 176 9/12/2022 11:58:22 AM


EQUILIBRIUM 177

nH2(g) + nI2(g) D 2nHI(g) (6.11)


Therefore, equilibrium constant for the N2(g) + O2(g) 2NO(g)
reaction is equal to Kc n. These findings are Solution
summarised in Table 6.4. It should be noted
that because the equilibrium constants Kc For the reaction equilibrium constant, Kc
and K ′c have different numerical values, it is can be written as,
2
important to specify the form of the balanced NO
Kc =
chemical equation when quoting the value of N 2 O2
an equilibrium constant. 2
2.8 10–3M
Table 6.4 Relations between Equilibrium =
Constants for a General Reaction 3.0 10 3 M 4.2 10 3 M
and its Multiples.
= 0.622
Chemical equation Equilibrium
constant
aA+bB c C + dD Kc
6.4 HOMOGENEOUS EQUILIBRIA
cC+dD aA+bB K′c =(1/Kc ) In a homogeneous system, all the reactants
and products are in the same phase.
na A + nb B ncC + ndD K′″c = (Kcn )
For example, in the gaseous reaction,
N 2(g) + 3H 2(g) 2NH 3(g), reactants and
products are in the homogeneous phase.
Problem 6.1
Similarly, for the reactions,
The following concentrations were CH3COOC2H5 (aq) + H2O (l) CH3COOH (aq)
obtained for the formation of NH3 from + C2H5OH (aq)
N 2 and H 2 at equilibrium at 500K.
[N2] = 1.5 × 10–2M. [H2] = 3.0 ×10–2 M and and, Fe3+ (aq) + SCN–(aq) Fe(SCN)2+ (aq)
[NH3] = 1.2 ×10–2M. Calculate equilibrium all the reactants and products are in
constant. homogeneous solution phase. We shall now
consider equilibrium constant for some
Solution
homogeneous reactions.
The equilibrium constant for the reaction,
N2(g) + 3H2(g) 2NH3(g) can be written as, 6.4.1 Equilibrium Constant in Gaseous
Systems
 NH3  g 
2
So far we have expressed equilibrium
Kc = constant of the reactions in terms of molar
 N 2  g  H2  g 
3
concentration of the reactants and products,
and used symbol, Kc for it. For reactions
=
1.2  10  2 2
involving gases, however, it is usually more
1.5  10  3.0  10 
2 2 3
convenient to express the equilibrium
constant in terms of partial pressure.
= 0.106 × 104 = 1.06 × 103 The ideal gas equation is written as,
pV = nRT
Problem 6.2
n
At equilibrium, the concentrations of ⇒ p = RT
N 2=3.0 × 10 –3M, O 2 = 4.2 × 10 –3M and V
NO= 2.8 × 10–3M in a sealed vessel at 800K. Here, p is the pressure in Pa, n is the number
What will be Kc for the reaction of moles of the gas, V is the volume in m3 and
T is the temperature in Kelvin

Rationalised 2023-24

Unit 6.indd 177 9/12/2022 11:58:22 AM


178 chemistry

Therefore,
n/V is concentration expressed in mol/m3  NH3 ( g ) [RT ]
2 −2

=  = K c ( RT )
−2
If concentration c, is in mol/L or mol/dm3,
 N 2 ( g ) H 2 ( g )
3
and p is in bar then
p = cRT,
or K p = K c ( RT ) (6.14)
−2
We can also write p = [gas]RT.
Here, R= 0.0831 bar litre/mol K Similarly, for a general reaction
At constant temperature, the pressure of aA + bB cC + dD
the gas is proportional to its concentration i.e.,
p ∝ [gas]
Kp =
( p )( p ) [C] [D] ( RT )(
C
c

=D
d c d c +d )

For reaction in equilibrium ( p )( p ) [ A ] [B] (RT )(


a
A
b
B
a b a +b )

H2(g) + I2(g) 2HI(g)


We can write either [ C ] [D]
c d

RT )
(c + d ) −(a +b )
b (
=
HI ( g )
2
[ A ] [B ]
a

Kc =
H2 ( g ) I2 ( g )
[ C ] [D]
c d

RT )
∆n
= K c ( RT )
∆n
b (
= (6.15)
or K c =
( p HI ) 2

(6.12)
[ A ] [B ]
a

( p )( p )
H2 I2
where ∆n = (number of moles of gaseous
products) – (number of moles of gaseous
Further, since p HI = HI ( g ) RT reactants) in the balanced chemical equation.
It is necessary that while calculating the value
pI2 = I2 ( g ) RT of Kp, pressure should be expressed in bar
p H2 = H2 ( g ) RT because standard state for pressure is 1 bar.
We know from Unit 1 that :
Therefore, 1pascal, Pa=1Nm–2, and 1bar = 105 Pa
HI ( g ) [RT ] Kp values for a few selected reactions at
( pHI )2
2 2

Kp = = different temperatures are given in Table 6.5


( p )( p )
H2 I2
H2 ( g ) RT . I2 ( g ) RT
Table 6.5 Equilibrium Constants, Kp for
HI ( g )
2
a Few Selected Reactions
= = Kc (6.13)
H2 ( g ) I2 ( g )
In this example, K p = K c i.e., both
equilibrium constants are equal. However,
this is not always the case. For example in
reaction
N2(g) + 3H2(g) 2NH3(g)

(p )
2
NH 3
Kp =
( p )( p )
3
N2 H2

Problem 6.3
 NH3 ( g ) [RT ]
2 2

= PCl5, PCl3 and Cl2 are at equilibrium at 500


 N 2 ( g ) RT . H 2 ( g ) (RT )
3 3
K and having concentration 1.59M PCl3,
1.59M Cl2 and 1.41 M PCl5.

Rationalised 2023-24

Unit 6.indd 178 11/30/2022 15:46:47


EQUILIBRIUM 179

Calculate Kc for the reaction, the value 0.194 should be neglected because
PCl5 PCl3 + Cl2 it will give concentration of the reactant
Solution which is more than initial concentration.
The equilibrium constant Kc for the above Hence the equilibrium concentrations are,
reaction can be written as, [CO2] = [H2-] = x = 0.067 M

Kc 
PCl  Cl   1.59
3 2
2

 1.79
[CO] = [H2O] = 0.1 – 0.067 = 0.033 M
PCl 5 1.41 Problem 6.5
Problem 6.4 For the equilibrium,
The value of K c = 4.24 at 800K for the 2NOCl(g) 2NO(g) + Cl2(g)
reaction, the value of the equilibrium constant, Kc is
CO (g) + H2O (g) CO2 (g) + H2 (g) 3.75 × 10 –6 at 1069 K. Calculate the Kp for
Calculate equilibrium concentrations of CO2, the reaction at this temperature?
H2, CO and H2O at 800 K, if only CO and
H2O are present initially at concentrations Solution
of 0.10M each.
We know that,
Solution Kp = Kc(RT)∆n
For the reaction, For the above reaction,
CO (g) + H2O (g) CO2 (g) + H2 (g) ∆n = (2+1) – 2 = 1
Kp = 3.75 ×10–6 (0.0831 × 1069)
Initial concentration:
Kp = 0.033
0.1M 0.1M 0 0
Let x mole per litre of each of the product
be formed. 6.5 HETEROGENEOUS EQUILIBRIA
At equilibrium: Equilibrium in a system having more than one
(0.1-x) M (0.1-x) M xM xM phase is called heterogeneous equilibrium.
The equilibrium between water vapour and
where x is the amount of CO2 and H2 at
equilibrium. liquid water in a closed container is an
example of heterogeneous equilibrium.
Hence, equilibrium constant can be written
as, H2O(l) H2O(g)
Kc = x2/(0.1-x)2 = 4.24 In this example, there is a gas phase and
x2 = 4.24(0.01 + x2-0.2x) a liquid phase. In the same way, equilibrium
x2 = 0.0424 + 4.24x2-0.848x between a solid and its saturated solution,
3.24x2 – 0.848x + 0.0424 = 0 Ca(OH)2 (s) + (aq) Ca2+ (aq) + 2OH–(aq)
a = 3.24, b = – 0.848, c = 0.0424 is a heterogeneous equilibrium.
(for quadratic equation ax2 + bx + c = 0, Heterogeneous equilibria often involve

x
 b  b2  4ac  pure solids or liquids. We can simplify
equilibrium expressions for the heterogeneous
2a equilibria involving a pure liquid or a pure
x = 0.848±√(0.848)2– 4(3.24)(0.0424)/ solid, as the molar concentration of a pure
solid or liquid is constant (i.e., independent
(3.24×2)
of the amount present). In other words if a
x = (0.848 ± 0.4118)/ 6.48
substance ‘X’ is involved, then [X(s)] and [X(l)]
x1 = (0.848 – 0.4118)/6.48 = 0.067 are constant, whatever the amount of ‘X’ is
x2 = (0.848 + 0.4118)/6.48 = 0.194 taken. Contrary to this, [X(g)] and [X(aq)] will

Rationalised 2023-24

Unit 6.indd 179 9/12/2022 11:58:24 AM


180 chemistry

vary as the amount of X in a given volume This shows that at a particular


varies. Let us take thermal dissociation of temperature, there is a constant concentration
calcium carbonate which is an interesting and or pressure of CO2 in equilibrium with CaO(s)
important example of heterogeneous chemical and CaCO3(s). Experimentally it has been
equilibrium. found that at 1100 K, the pressure of CO2
in equilibrium with CaCO3(s) and CaO(s), is
CaCO3 (s) CaO (s) + CO2 (g) (6.16) 2.0 ×105 Pa. Therefore, equilibrium constant
On the basis of the stoichiometric equation, at 1100K for the above reaction is:
we can write, Kp = PCO2 = 2 × 105 Pa/105 Pa = 2.00
CaO  s  CO2  g  Similarly, in the equilibrium between
Kc 
CaCO3  s  nickel, carbon monoxide and nickel carbonyl
(used in the purification of nickel),
Since [CaCO3(s)] and [CaO(s)] are both
constant, therefore modified equilibrium Ni (s) + 4 CO (g) Ni(CO)4 (g),
constant for the thermal decomposition of the equilibrium constant is written as
calcium carbonate will be
 Ni CO 4 
K´c = [CO2(g)] (6.17) Kc  
or Kp = pCO (6.18) CO4
2

It must be remembered that for the


existence of heterogeneous equilibrium pure
Units of Equilibrium Constant solids or liquids must also be present (however
The value of equilibrium constant Kc can be small the amount may be) at equilibrium, but
calculated by substituting the concentration their concentrations or partial pressures do
terms in mol/L and for Kp partial pressure not appear in the expression of the equilibrium
is substituted in Pa, kPa, bar or atm. This constant. In the reaction,
results in units of equilibrium constant based Ag2O(s) + 2HNO3(aq) 2AgNO3(aq) +H2O(l)
on molarity or pressure, unless the exponents
of both the numerator and denominator are
 AgNO 
2
3
same. Kc =
HNO 
2
For the reactions, 3
H2(g) + I2(g) 2HI, Kc and Kp have no
unit.
N2O4(g) 2NO2 (g), Kc has unit mol/L
and Kp has unit bar Problem 6.6
Equilibrium constants can also be The value of Kp for the reaction,
expressed as dimensionless quantities if the CO2 (g) + C (s) 2CO (g)
standard state of reactants and products is 3.0 at 1000 K. If initially PCO = 0.48
2
are specified. For a pure gas, the standard bar and PCO = 0 bar and pure graphite is
state is 1bar. Therefore a pressure of 4 bar present, calculate the equilibrium partial
in standard state can be expressed as 4 pressures of CO and CO2.
bar/1 bar = 4, which is a dimensionless
number. Standard state (c0) for a solute is 1 Solution
molar solution and all concentrations can be For the reaction,
measured with respect to it. The numerical
value of equilibrium constant depends on the
let ‘x’ be the decrease in pressure of CO2,
standard state chosen. Thus, in this system
then
both Kp and Kc are dimensionless quantities CO2(g) + C(s) 2CO(g)
but have different numerical values due to Initial
different standard states. pressure: 0.48 bar 0

Rationalised 2023-24

Unit 6.indd 180 9/12/2022 11:58:24 AM


EQUILIBRIUM 181

5. The equilibrium constant K for a reaction


At equilibrium: is related to the equilibrium constant
(0.48 – x)bar 2x bar of the corresponding reaction, whose
equation is obtained by multiplying or
pC2 O dividing the equation for the original
Kp =
pC O2 reaction by a small integer.

Kp = (2x)2/(0.48 – x) = 3 Let us consider applications of equilibrium


constant to:
4x2 = 3(0.48 – x)
• predict the extent of a reaction on the
4x2 = 1.44 – x basis of its magnitude,
4x2 + 3x – 1.44 = 0
• predict the direction of the reaction, and
a = 4, b = 3, c = –1.44

b  
calculate equilibrium concentrations.
b2  4ac
x 6.6.1 Predicting the Extent of a
2a Reaction
= [–3 ± √(3)2– 4(4)(–1.44)]/2 × 4 The numerical value of the equilibrium
= (–3 ± 5.66)/8 constant for a reaction indicates the extent
of the reaction. But it is important to note
= (–3 + 5.66)/8 (as value of x cannot be that an equilibrium constant does not give
negative hence we neglect that value)
any information about the rate at which
x = 2.66/8 = 0.33 the equilibrium is reached. The magnitude
The equilibrium partial pressures are, of Kc or Kp is directly proportional to the
concentrations of products (as these appear
pCO = 2x = 2 × 0.33 = 0.66 bar
2 in the numerator of equilibrium constant
pCO = 0.48 – x = 0.48 – 0.33 = 0.15 bar expression) and inversely proportional to the
2
concentrations of the reactants (these appear
6.6 APPLICATIONS OF EQUILIBRIUM in the denominator). This implies that a high
CONSTANTS value of K is suggestive of a high concentration
of products and vice-versa.
Before considering the applications of
equilibrium constants, let us summarise the We can make the following generalisations
important features of equilibrium constants concerning the composition of equilibrium
as follows: mixtures:
1. Expression for equilibrium constant is • If Kc > 103, products predominate over
applicable only when concentrations of reactants, i.e., if Kc is very large, the
the reactants and products have attained reaction proceeds nearly to completion.
constant value at equilibrium state. Consider the following examples:
2. The value of equilibrium constant is (a) The reaction of H 2 with O 2 at 500 K
independent of initial concentrations of has a very large equilibrium constant,
the reactants and products. Kc = 2.4 × 1047.
3. Equilibrium constant is temperature (b) H2(g) + Cl 2(g) 2HCl(g) at 300K has
dependent having one unique value for Kc = 4.0 × 1031.
a particular reaction represented by a (c) H 2(g) + Br 2(g) 2HBr (g) at 300 K,
balanced equation at a given temperature. Kc = 5.4 × 1018
4. The equilibrium constant for the reverse
• If Kc < 10–3, reactants predominate over
reaction is equal to the inverse of the
products, i.e., if Kc is very small, the
equilibrium constant for the forward
reaction proceeds rarely. Consider the
reaction. following examples:

Rationalised 2023-24

Unit 6.indd 181 9/12/2022 11:58:24 AM


182 chemistry

(a) The decomposition of H2O into H2 and If Qc = Kc, the reaction mixture is already
O2 at 500 K has a very small equilibrium at equilibrium.
constant, Kc = 4.1 × 10 –48 Consider the gaseous reaction of H 2
(b) N2(g) + O2(g) 2NO(g), with I2,
at 298 K has Kc = 4.8 ×10 – 31. H2(g) + I2(g) 2HI(g); Kc = 57.0 at 700 K.
• If K c is in the range of 10 – 3 to 10 3 , Suppose we have molar concentrations
appreciable concentrations of both [H2]t=0.10M, [I2]t = 0.20 M and [HI]t = 0.40 M.
reactants and products are present. (the subscript t on the concentration symbols
Consider the following examples: means that the concentrations were measured
(a) For reaction of H2 with I2 to give HI, at some arbitrary time t, not necessarily at
equilibrium).
Kc = 57.0 at 700K.
Thus, the reaction quotient, Qc at this
(b) Also, gas phase decomposition of N2O4
stage of the reaction is given by,
to NO2 is another reaction with a value
Qc = [HI]t2 / [H2]t [I2]t = (0.40)2/ (0.10)×(0.20)
of K c = 4.64 × 10 –3 at 25°C which is
neither too small nor too large. Hence, = 8.0
equilibrium mixtures contain appreciable Now, in this case, Qc (8.0) does not equal
concentrations of both N2O4 and NO2. Kc (57.0), so the mixture of H2(g), I2(g) and HI(g)
These generarlisations are illustrated in is not at equilibrium; that is, more H2(g) and
Fig. 6.6 I2(g) will react to form more HI(g) and their
concentrations will decrease till Qc = Kc.
The reaction quotient, Q c is useful in
predicting the direction of reaction by
comparing the values of Qc and Kc.
Thus, we can make the following
generalisations concerning the direction of
Fig.6.6 Dependence of extent of reaction on Kc the reaction (Fig. 6.7) :

6.6.2 Predicting the Direction of the


Reaction
The equilibrium constant helps in predicting
the direction in which a given reaction will
proceed at any stage. For this purpose,
we calculate the reaction quotient Q.
The reaction quotient, Q (Q c with molar
concentrations and QP with partial pressures) Fig. 6.7 Predicting the direction of the reaction
is defined in the same way as the equilibrium • If Qc < Kc, net reaction goes from left to
constant Kc except that the concentrations right
in Qc are not necessarily equilibrium values. • If Qc > Kc, net reaction goes from right to
For a general reaction: left.
aA+bB c C + d D (6.19) • If Qc = Kc, no net reaction occurs.
Qc = [C]c[D]d / [A]a[B]b (6.20)
Problem 6.7
Then,
The value of Kc for the reaction
If Qc > Kc, the reaction will proceed in the 2A B + C is 2 × 10–3. At a given time,
direction of reactants (reverse reaction). the composition of reaction mixture is
If Qc < Kc, the reaction will proceed in the [A] = [B] = [C] = 3 × 10–4 M. In which direction
the reaction will proceed?
direction of the products (forward reaction).

Rationalised 2023-24

Unit 6.indd 182 9/12/2022 11:58:25 AM


EQUILIBRIUM 183

Solution The total pressure at equilbrium was


For the reaction the reaction quotient Qc is found to be 9.15 bar. Calculate Kc, Kp and
given by, partial pressure at equilibrium.
Qc = [B][C]/ [A]2 Solution
as [A] = [B] = [C] = 3 × 10–4M
Qc = (3 ×10–4)(3 × 10 –4) / (3 ×10 –4)2 = 1 We know pV = nRT
as Qc > Kc so the reaction will proceed in the Total volume (V ) = 1 L
reverse direction.
Molecular mass of N2O4 = 92 g
6.6.3 Calculating Equilibrium Number of moles = 13.8g/92 g = 0.15
Concentrations of the gas (n)
In case of a problem in which we know the Gas constant (R) = 0.083 bar L mol–1K–1
initial concentrations but do not know any of Temperature (T ) = 400 K
the equilibrium concentrations, the following
three steps shall be followed: pV = nRT
Step 1. Write the balanced equation for the p × 1L = 0.15 mol × 0.083 bar L mol–1K–1
reaction. × 400 K
Step 2. Under the balanced equation, make p = 4.98 bar
a table that lists for each substance involved N2O4 2NO2
in the reaction: Initial pressure: 4.98 bar 0
(a) the initial concentration, At equilibrium: (4.98 – x) bar 2x bar
(b) the change in concentration on going to Hence,
equilibrium, and ptotal at equilibrium = pN + pNO
2O4 2
(c) the equilibrium concentration.
9.15 = (4.98 – x) + 2x
In constructing the table, define x as the
concentration (mol/L) of one of the substances 9.15 = 4.98 + x
that reacts on going to equilibrium, then use x = 9.15 – 4.98 = 4.17 bar
the stoichiometry of the reaction to determine Partial pressures at equilibrium are,
the concentrations of the other substances in
pN = 4.98 – 4.17 = 0.81bar
terms of x. 2O4
pNO = 2x = 2 × 4.17 = 8.34 bar
Step 3. Substitute the equilibrium 2

 
2
concentrations into the equilibrium equation K p = p NO2 / p N 2O4
for the reaction and solve for x. If you are
= (8.34)2/0.81 = 85.87
to solve a quadratic equation choose the
mathematical solution that makes chemical Kp = Kc(RT)∆n
sense. 85.87 = Kc(0.083 × 400)1
Step 4. Calculate the equilibrium Kc = 2.586 = 2.6
concentrations from the calculated value of x.
Step 5. Check your results by substituting Problem 6.9
them into the equilibrium equation. 3.00 mol of PCl5 kept in 1L closed reaction
vessel was allowed to attain equilibrium
at 380K. Calculate composition of the
Problem 6.8 mixture at equilibrium. Kc= 1.80
13.8g of N2O4 was placed in a 1L reaction Solution
vessel at 400K and allowed to attain PCl5 PCl3 + Cl2
equilibrium Initial
N 2O4 (g) 2NO2 (g) concentration: 3.0 0 0

Rationalised 2023-24

Unit 6.indd 183 9/12/2022 11:58:25 AM


184 chemistry

Taking antilog of both sides, we get,


Let x mol per litre of PCl5 be dissociated,
At equilibrium: K = e–∆G/RT (6.23)
(3-x) x x Hence, using the equation (6.23), the
Kc = [PCl3][Cl2]/[PCl5] reaction spontaneity can be interpreted in
terms of the value of ∆G .
1.8 = x2/ (3 – x)
• If ∆G  < 0, then –∆G /RT is positive,
x2 + 1.8x – 5.4 = 0
and e –∆DG /RT>1, making K >1, which
x = [–1.8 ± √(1.8)2 – 4(–5.4)]/2 implies a spontaneous reaction or the
x = [–1.8 ± √3.24 + 21.6]/2 reaction which proceeds in the forward
direction to such an extent that the
x = [–1.8 ± 4.98]/2
products are present predominantly.
x = [–1.8 + 4.98]/2 = 1.59
• If ∆G  > 0, then –∆G /RT is negative, and
[PCl5] = 3.0 – x = 3 –1.59 = 1.41 M
e –∆G </RT 1, that is , K < 1, which implies

[PCl3] = [Cl2] = x = 1.59 M a non-spontaneous reaction or a reaction


which proceeds in the forward direction
6.7 Relationship between to such a small degree that only a very
Equilibrium Constant K, minute quantity of product is formed.
Reaction Quotient Q and
Gibbs Energy G Problem 6.10
The value of Kc for a reaction does not depend The value of ∆G  for the phosphorylation of
on the rate of the reaction. However, as you glucose in glycolysis is 13.8 kJ/mol. Find
have studied in Unit 5, it is directly related the value of Kc at 298 K.
to the thermodynamics of the reaction and
in particular, to the change in Gibbs energy, Solution
∆G. If, ∆G  = 13.8 kJ/mol = 13.8 × 103J/mol
• ∆G is negative, then the reaction is Also, ∆G  = – RT lnKc
spontaneous and proceeds in the forward
direction. Hence, ln Kc = –13.8 × 103J/mol
(8.314 J mol –1K –1 × 298 K)
• ∆G is positive, then reaction is considered
non-spontaneous. Instead, as reverse ln Kc = – 5.569
reaction would have a negative ∆G, the Kc = e–5.569
products of the forward reaction shall be Kc = 3.81 × 10 –3
converted to the reactants.
Problem 6.11
• ∆G is 0, reaction has achieved equilibrium;
at this point, there is no longer any free Hydrolysis of sucrose gives,
energy left to drive the reaction. Sucrose + H2O Glucose + Fructose
A mathematical expression of this Equilibrium constant Kc for the reaction is
thermodynamic view of equilibrium can be 2 ×1013 at 300K. Calculate ∆G  at 300K.
described by the following equation:
Solution
∆G = ∆G + RT lnQ (6.21)
∆G  = – RT lnKc
where, G is standard Gibbs energy.

∆G  = – 8.314J mol–1K–1×
At equilibrium, when ∆G = 0 and Q = Kc, 300K × ln(2×1013)
the equation (6.21) becomes, ∆G  = – 7.64 ×104 J mol–1
∆G = ∆G + RT ln K = 0
6.8 FACTORS AFFECTING EQUILIBRIA
∆G = – RT lnK (6.22)
One of the principal goals of chemical
lnK = – ∆G / RT
synthesis is to maximise the conversion of the

Rationalised 2023-24

Unit 6.indd 184 9/12/2022 11:58:25 AM


EQUILIBRIUM 185

reactants to products while minimising the “When the concentration of any of the
expenditure of energy. This implies maximum reactants or products in a reaction at
yield of products at mild temperature and equilibrium is changed, the composition
pressure conditions. If it does not happen, of the equilibrium mixture changes so as
then the experimental conditions need to be to minimize the effect of concentration
adjusted. For example, in the Haber process changes”.
for the synthesis of ammonia from N2 and Let us take the reaction,
H2, the choice of experimental conditions is
of real economic importance. Annual world H2(g) + I2(g) 2HI(g)
production of ammonia is about hundred If H2 is added to the reaction mixture
million tones, primarily for use as fertilisers. at equilibrium, then the equilibrium of the
Equilibrium constant, Kc is independent reaction is disturbed. In order to restore it,
of initial concentrations. But if a system at the reaction proceeds in a direction wherein
equilibrium is subjected to a change in the H2 is consumed, i.e., more of H2 and I2 react
concentration of one or more of the reacting to form HI and finally the equilibrium shifts
substances, then the system is no longer at in right (forward) direction (Fig.6.8). This is in
equilibrium; and net reaction takes place in accordance with the Le Chatelier’s principle
some direction until the system returns to which implies that in case of addition of a
equilibrium once again. Similarly, a change reactant/product, a new equilibrium will
in temperature or pressure of the system may be set up in which the concentration of the
also alter the equilibrium. In order to decide reactant/product should be less than what it
what course the reaction adopts and make was after the addition but more than what it
a qualitative prediction about the effect of a was in the original mixture.
change in conditions on equilibrium we use
Le Chatelier’s principle. It states that a
change in any of the factors that determine
the equilibrium conditions of a system
will cause the system to change in such a
manner so as to reduce or to counteract
the effect of the change. This is applicable
to all physical and chemical equilibria.
We shall now be discussing factors which
can influence the equilibrium.
6.8.1 Effect of Concentration Change
In general, when equilibrium is disturbed
by the addition/removal of any reactant/
products, Le Chatelier’s principle predicts
that:
• The concentration stress of an added
reactant/product is relieved by net
reaction in the direction that consumes Fig. 6.8 Effect of addition of H 2 on change
of concentration for the reactants
the added substance. and products in the reaction,
• The concentration stress of a removed H2(g) + I2 (g) 2HI(g)
reactant/product is relieved by net
reaction in the direction that replenishes The same point can be explained in terms
the removed substance. of the reaction quotient, Qc,
or in other words, Qc = [HI]2/ [H2][I2]

Rationalised 2023-24

Unit 6.indd 185 11/30/2022 15:48:33


186 chemistry

Addition of hydrogen at equilibrium concentration of [Fe(SCN)]2+ decreases, the


results in value of Qc being less than Kc . Thus, intensity of red colour decreases.
in order to attain equilibrium again reaction Addition of aq. HgCl2 also decreases red
moves in the forward direction. Similarly, colour because Hg2+ reacts with SCN– ions to
we can say that removal of a product also form stable complex ion [Hg(SCN)4]2–. Removal
boosts the forward reaction and increases of free SCN – (aq) shifts the equilibrium
the concentration of the products and this in equation (6.24) from right to left to
has great commercial application in cases replenish SCN – ions. Addition of potassium
of reactions, where the product is a gas or a thiocyanate on the other hand increases the
volatile substance. In case of manufacture of colour intensity of the solution as it shift the
ammonia, ammonia is liquified and removed equilibrium to right.
from the reaction mixture so that reaction
keeps moving in forward direction. Similarly, 6.8.2 Effect of Pressure Change
in the large scale production of CaO (used A pressure change obtained by changing the
as important building material) from CaCO3, volume can affect the yield of products in
constant removal of CO2 from the kiln drives case of a gaseous reaction where the total
the reaction to completion. It should be number of moles of gaseous reactants and
remembered that continuous removal of a total number of moles of gaseous products are
product maintains Qc at a value less than Kc different. In applying Le Chatelier’s principle
and reaction continues to move in the forward to a heterogeneous equilibrium the effect
direction. of pressure changes on solids and liquids
can be ignored because the volume (and
Effect of Concentration – An experiment concentration) of a solution/liquid is nearly
This can be demonstrated by the following independent of pressure.
reaction: Consider the reaction,
Fe3+(aq)+ SCN –(aq) [Fe(SCN)]2+(aq) (6.24)
CO(g) + 3H2(g) CH4(g) + H2O(g)
yellow colourless deep red
Here, 4 mol of gaseous reactants (CO + 3H2)
become 2 mol of gaseous products (CH4 +
H2O). Suppose equilibrium mixture (for above
(6.25) reaction) kept in a cylinder fitted with a piston
at constant temperature is compressed to
A reddish colour appears on adding two
one half of its original volume. Then, total
drops of 0.002 M potassium thiocynate solution
pressure will be doubled (according to
to 1 mL of 0.2 M iron(III) nitrate solution due
pV = constant). The partial pressure and
to the formation of [Fe(SCN)]2+. The intensity therefore, concentration of reactants and
of the red colour becomes constant on products have changed and the mixture is no
attaining equilibrium. This equilibrium can be longer at equilibrium. The direction in which
shifted in either forward or reverse directions the reaction goes to re-establish equilibrium
depending on our choice of adding a reactant can be predicted by applying the Le Chatelier’s
or a product. The equilibrium can be shifted principle. Since pressure has doubled,
in the opposite direction by adding reagents the equilibrium now shifts in the forward
that remove Fe3+ or SCN – ions. For example, direction, a direction in which the number
oxalic acid (H2C2O4), reacts with Fe3+ ions of moles of the gas or pressure decreases (we
to form the stable complex ion [Fe(C2O4)3]3–, know pressure is proportional to moles of the
thus decreasing the concentration of free gas). This can also be understood by using
Fe3+(aq). In accordance with the Le Chatelier’s reaction quotient, Qc. Let [CO], [H2], [CH4]
principle, the concentration stress of removed and [H2O] be the molar concentrations at
Fe3+ is relieved by dissociation of [Fe(SCN)]2+ equilibrium for methanation reaction. When
to replenish the Fe 3+ ions. Because the volume of the reaction mixture is halved, the

Rationalised 2023-24

Unit 6.indd 186 9/12/2022 11:58:25 AM


EQUILIBRIUM 187

partial pressure and the concentration are Production of ammonia according to the
doubled. We obtain the reaction quotient by reaction,
replacing each equilibrium concentration by N2(g) + 3H2(g) 2NH3(g);
double its value.
∆H= – 92.38 kJ mol–1
CH4 ( g ) H2O ( g )
Qc =  is an exothermic process. According to
CO ( g ) H2 ( g )
3
Le Chatelier’s principle, raising the
temperature shifts the equilibrium to left
As Qc < Kc , the reaction proceeds in the and decreases the equilibrium concentration
forward direction. of ammonia. In other words, low temperature
In reaction C(s) + CO2(g) 2CO(g), when is favourable for high yield of ammonia, but
practically very low temperatures slow down
pressure is increased, the reaction goes in the
the reaction and thus a catalyst is used.
reverse direction because the number of moles
of gas increases in the forward direction. Effect of Temperature – An experiment
Effect of temperature on equilibrium can
6.8.3 Effect of Inert Gas Addition
be demonstrated by taking NO2 gas (brown
If the volume is kept constant and an inert gas in colour) which dimerises into N 2O 4 gas
such as argon is added which does not take (colourless).
part in the reaction, the equilibrium remains
2NO2(g) N2O4(g); ∆H = –57.2 kJ mol–1
undisturbed. It is because the addition of
an inert gas at constant volume does not NO 2 gas prepared by addition of Cu
change the partial pressures or the molar turnings to conc. HNO3 is collected in two
concentrations of the substance involved in 5 mL test tubes (ensuring same intensity
the reaction. The reaction quotient changes of colour of gas in each tube) and stopper
only if the added gas is a reactant or product sealed with araldite. Three 250 mL beakers
involved in the reaction. 1, 2 and 3 containing freesing mixture, water
at room temperature and hot water (363K),
6.8.4 Effect of Temperature Change respectively, are taken (Fig. 6.9). Both the test
Whenever an equilibrium is disturbed by tubes are placed in beaker 2 for 8-10 minutes.
a change in the concentration, pressure or After this one is placed in beaker 1 and the
volume, the composition of the equilibrium other in beaker 3. The effect of temperature
mixture changes because the reaction on direction of reaction is depicted very well
quotient, Qc no longer equals the equilibrium in this experiment. At low temperatures in
constant, Kc. However, when a change in beaker 1, the forward reaction of formation of
temperature occurs, the value of equilibrium N2O4 is preferred, as reaction is exothermic,
constant, Kc is changed. and thus, intensity of brown colour due
to NO2 decreases. While in beaker 3, high
In general, the temperature dependence temperature favours the reverse reaction of
of the equilibrium constant depends on the
sign of ∆H for the reaction.
• The equilibrium constant for an exothermic
reaction (negative ∆H) decreases as the
temperature increases.
• The equilibrium constant for an
endothermic reaction (positive ∆H)
increases as the temperature increases.
Temperature changes affect the Fig. 6.9 Effect of temperature on equilibrium for
equilibrium constant and rates of reactions. the reaction, 2NO2 (g) N2O4 (g)

Rationalised 2023-24

Unit 6.indd 187 11/2/2022 4:11:28 PM


188 chemistry

formation of NO2 and thus, the brown colour Similarly, in manufacture of sulphuric
intensifies. acid by contact process,
Effect of temperature can also be seen in 2SO2(g) + O2(g) 2SO3(g); Kc = 1.7 × 1026
an endothermic reaction, though the value of K is suggestive of reaction
[Co(H2O) 6]3+ (aq) + 4Cl –(aq) [CoCl4]2–(aq) + going to completion, but practically the
6H2O(l) oxidation of SO2 to SO3 is very slow. Thus,
pink colourless blue platinum or divanadium penta-oxide (V2O5)
is used as catalyst to increase the rate of the
At room temperature, the equilibrium
mixture is blue due to [CoCl4]2–. When cooled reaction.
in a freesing mixture, the colour of the mixture Note: If a reaction has an exceedingly small
turns pink due to [Co(H2O)6]3+. K, a catalyst would be of little help.

6.8.5 Effect of a Catalyst 6.9 IONIC EQUILIBRIUM IN SOLUTION


A catalyst increases the rate of the chemical Under the effect of change of concentration
reaction by making available a new low energy on the direction of equilibrium, you have
pathway for the conversion of reactants to incidently come across with the following
products. It increases the rate of forward equilibrium which involves ions:
and reverse reactions that pass through the Fe3+(aq) + SCN–(aq) [Fe(SCN)]2+(aq)
same transition state and does not affect There are numerous equilibria that involve
equilibrium. Catalyst lowers the activation ions only. In the following sections we will
energy for the forward and reverse reactions study the equilibria involving ions. It is well
by exactly the same amount. Catalyst does known that the aqueous solution of sugar
not affect the equilibrium composition of does not conduct electricity. However, when
a reaction mixture. It does not appear in
common salt (sodium chloride) is added
the balanced chemical equation or in the
to water it conducts electricity. Also, the
equilibrium constant expression.
conductance of electricity increases with an
Let us consider the formation of NH3 increase in concentration of common salt.
from dinitrogen and dihydrogen which is Michael Faraday classified the substances
highly exothermic reaction and proceeds into two categories based on their ability
with decrease in total number of moles to conduct electricity. One category of
formed as compared to the reactants. substances conduct electricity in their
Equilibrium constant decreases with increase aqueous solutions and are called electrolytes
in temperature. At low temperature rate while the other do not and are thus, referred to
decreases and it takes long time to reach at as non-electrolytes. Faraday further classified
equilibrium, whereas high temperatures give electrolytes into strong and weak electrolytes.
satisfactory rates but poor yields. Strong electrolytes on dissolution in water
German chemist, Fritz Haber discovered are ionized almost completely, while the weak
that a catalyst consisting of iron catalyse electrolytes are only partially dissociated.
the reaction to occur at a satisfactory rate For example, an aqueous solution of
at temperatures, where the equilibrium sodium chloride is comprised entirely of
concentration of NH3 is reasonably favourable. sodium ions and chloride ions, while that
Since the number of moles formed in the of acetic acid mainly contains unionized
reaction is less than those of reactants, the acetic acid molecules and only some acetate
yield of NH3 can be improved by increasing ions and hydronium ions. This is because
the pressure. there is almost 100% ionization in case
Optimum conditions of temperature of sodium chloride as compared to less
and pressure for the synthesis of NH3 using than 5% ionization of acetic acid which is
catalyst are around 500 °C and 200 atm. a weak electrolyte. It should be noted

Rationalised 2023-24

Unit 6.indd 188 11/2/2022 4:11:28 PM


EQUILIBRIUM 189

that in weak electrolytes, equilibrium is exists in solid state as a cluster of positively


established between ions and the unionized charged sodium ions and negatively charged
molecules. This type of equilibrium involving chloride ions which are held together due to
ions in aqueous solution is called ionic electrostatic interactions between oppositely
equilibrium. Acids, bases and salts come charged species (Fig.6.10). The electrostatic
under the category of electrolytes and may act forces between two charges are inversely
as either strong or weak electrolytes. proportional to dielectric constant of the
medium. Water, a universal solvent, possesses
6.10 ACIDS, BASES AND SALTS a very high dielectric constant of 80. Thus,
Acids, bases and salts find widespread when sodium chloride is dissolved in water,
occurrence in nature. Hydrochloric acid the electrostatic interactions are reduced by
present in the gastric juice is secreted by the a factor of 80 and this facilitates the ions to
lining of our stomach in a significant amount move freely in the solution. Also, they are
of 1.2-1.5 L/day and is essential for digestive well-separated due to hydration with water
processes. Acetic acid is known to be the main molecules.
constituent of vinegar. Lemon and orange
juices contain citric and ascorbic acids, and
tartaric acid is found in tamarind paste. As
most of the acids taste sour, the word “acid”
has been derived from a latin word “acidus”
meaning sour. Acids are known to turn blue
litmus paper into red and liberate dihydrogen
on reacting with some metals. Similarly, bases
are known to turn red litmus paper blue, taste
bitter and feel soapy. A common example
of a base is washing soda used for washing
purposes. When acids and bases are mixed
in the right proportion they react with each
Fig.6.10 Dissolution of sodium chloride in water.
other to give salts. Some commonly known
Na+ and Cl – ions are stablised by their
examples of salts are sodium chloride, barium hydration with polar water molecules.
sulphate, sodium nitrate. Sodium chloride
(common salt) is an important component of Comparing, the ionization of hydrochloric
our diet and is formed by reaction between acid with that of acetic acid in water we find
hydrochloric acid and sodium hydroxide. It that though both of them are polar covalent

Faraday was born near London into a family of very limited means. At the age of
14 he was an apprentice to a kind bookbinder who allowed Faraday to read the
books he was binding. Through a fortunate chance he became laboratory assistant
to Davy, and during 1813-4, Faraday accompanied him to the Continent. During
this trip he gained much from the experience of coming into contact with many of the
leading scientists of the time. In 1825, he succeeded Davy as Director of the Royal
Institution laboratories, and in 1833 he also became the first Fullerian Professor of
Chemistry. Faraday’s first important work was on analytical chemistry. After 1821
Michael Faraday
much of his work was on electricity and magnetism and different electromagnetic
(1791–1867)
phenomena. His ideas have led to the establishment of modern field theory.
He discovered his two laws of electrolysis in 1834. Faraday was a very modest and kind hearted
person. He declined all honours and avoided scientific controversies. He preferred to work alone and
never had any assistant. He disseminated science in a variety of ways including his Friday evening
discourses, which he founded at the Royal Institution. He has been very famous for his Christmas
lecture on the ‘Chemical History of a Candle’. He published nearly 450 scientific papers.

Rationalised 2023-24

Unit 6.indd 189 9/12/2022 11:58:26 AM


190 chemistry

molecules, former is completely ionized into


its constituent ions, while the latter is only Hydronium and Hydroxyl Ions
partially ionized (< 5%). The extent to which Hydrogen ion by itself is a bare proton with very
ionization occurs depends upon the strength small size (~10–15 m radius) and intense electric
of the bond and the extent of solvation field, binds itself with the water molecule at
of ions produced. The terms dissociation one of the two available lone pairs on it giving
and ionization have earlier been used with H3O+. This species has been detected in many
different meaning. Dissociation refers to the compounds (e.g., H3O+Cl–) in the solid state. In
process of separation of ions in water already aqueous solution the hydronium ion is further
existing as such in the solid state of the solute, hydrated to give species like H5O2+, H7O3+ and
as in sodium chloride. On the other hand, H9O4+. Similarly the hydroxyl ion is hydrated
to give several ionic species like H3O2–, H5O3–
ionization corresponds to a process in which
and H7O4– etc.
a neutral molecule splits into charged ions in
the solution. Here, we shall not distinguish
between the two and use the two terms
interchangeably.

6.10.1 Arrhenius Concept of Acids and


Bases
According to Arrhenius theory, acids are H9O4+
substances that dissociates in water to
give hydrogen ions H +(aq) and bases are 6.10.2 The Brönsted-Lowry Acids and
substances that produce hydroxyl ions Bases
OH –(aq). The ionization of an acid HX (aq) can The Danish chemist, Johannes Brönsted and
be represented by the following equations: the English chemist, Thomas M. Lowry gave
HX (aq) → H+(aq) + X– (aq) a more general definition of acids and bases.
or According to Brönsted-Lowry theory, acid
HX(aq) + H2O(l) → H3O+(aq) + X –(aq) is a substance that is capable of donating a
hydrogen ion H+ and bases are substances
A bare proton, H+ is very reactive and
capable of accepting a hydrogen ion, H+. In
cannot exist freely in aqueous solutions.
short, acids are proton donors and bases are
Thus, it bonds to the oxygen atom of a solvent
proton acceptors.
water molecule to give trigonal pyramidal
hydronium ion, H3O+ {[H (H2O)]+} (see box). In Consider the example of dissolution of NH3
this chapter we shall use H+(aq) and H3O+(aq) in H2O represented by the following equation:
interchangeably to mean the same i.e., a
hydrated proton.
Similarly, a base molecule like MOH
ionizes in aqueous solution according to the
equation:
MOH(aq) → M+(aq) + OH–(aq)
The hydroxyl ion also exists in the hydrated
form in the aqueous solution. Arrhenius
concept of acid and base, however, suffers The basic solution is formed due to the
from the limitation of being applicable only to presence of hydroxyl ions. In this reaction,
aqueous solutions and also, does not account water molecule acts as proton donor and
for the basicity of substances like, ammonia ammonia molecule acts as proton acceptor
which do not possess a hydroxyl group. and are thus, called Lowry-Brönsted acid and

Rationalised 2023-24

Unit 6.indd 190 9/12/2022 11:58:26 AM


EQUILIBRIUM 191

Arrhenius was born near Uppsala, Sweden. He presented his thesis, on the
conductivities of electrolyte solutions, to the University of Uppsala in 1884. For the
next five years he travelled extensively and visited a number of research centers
in Europe. In 1895 he was appointed professor of physics at the newly formed
University of Stockholm, serving its rector from 1897 to 1902. From 1905 until his
death he was Director of physical chemistry at the Nobel Institute in Stockholm. He
continued to work for many years on electrolytic solutions. In 1899 he discussed
the temperature dependence of reaction rates on the basis of an equation, now
usually known as Arrhenius equation.
He worked in a variety of fields, and made important contributions to
Svante Arrhenius
immunochemistry, cosmology, the origin of life, and the causes of ice age. He was
(1859-1927)
the first to discuss the ‘green house effect’ calling by that name. He received Nobel
Prize in Chemistry in 1903 for his theory of electrolytic dissociation and its use in
the development of chemistry.

base, respectively. In the reverse reaction, in case of ammonia it acts as an acid by


H+ is transferred from NH4+ to OH–. In this donating a proton.
case, NH 4+ acts as a Bronsted acid while
Problem 6.12
OH– acted as a Brönsted base. The acid-base
pair that differs only by one proton is called What will be the conjugate bases for the
a conjugate acid-base pair. Therefore, OH – following Brönsted acids: HF, H2SO4 and
HCO3– ?
is called the conjugate base of an acid H2O
and NH4+ is called conjugate acid of the base Solution
NH3. If Brönsted acid is a strong acid then The conjugate bases should have one
its conjugate base is a weak base and vice- proton less in each case and therefore the
versa. It may be noted that conjugate acid corresponding conjugate bases are: F –,
has one extra proton and each conjugate base HSO4– and CO32– respectively.
has one less proton.
Problem 6.13
Consider the example of ionization of
Write the conjugate acids for the following
hydrochloric acid in water. HCl(aq) acts as Brönsted bases: NH2–, NH3 and HCOO–.
an acid by donating a proton to H2O molecule
which acts as a base. Solution
The conjugate acid should have one extra
proton in each case and therefore the
corresponding conjugate acids are: NH3,
NH4+ and HCOOH respectively.
Problem 6.14
The species: H2O, HCO3–, HSO4– and NH3 can
act both as Bronsted acids and bases. For
each case give the corresponding conjugate
It can be seen in the above equation, that acid and conjugate base.
water acts as a base because it accepts the
Solution
proton. The species H3O+ is produced when
water accepts a proton from HCl. Therefore, The answer is given in the following Table:
Cl – is a conjugate base of HCl and HCl is the Species Conjugate Conjugate
conjugate acid of base Cl –. Similarly, H2O is acid base
a conjugate base of an acid H3O+ and H3O+ is
H2O H3O+ OH –
a conjugate acid of base H2O.
HCO3– H2CO3 CO32–
It is interesting to observe the dual role
HSO4– H2SO4 SO42–
of water as an acid and a base. In case of
reaction with HCl water acts as a base while NH3 NH4+ NH2–

Rationalised 2023-24

Unit 6.indd 191 9/12/2022 11:58:26 AM


192 chemistry

6.10.3 Lewis Acids and Bases (HClO4), hydrochloric acid (HCl), hydrobromic
G.N. Lewis in 1923 defined an acid as a acid (HBr), hyrdoiodic acid (HI), nitric acid
species which accepts electron pair and base (HNO3) and sulphuric acid (H2SO4) are termed
which donates an electron pair. As far as bases strong because they are almost completely
are concerned, there is not much difference dissociated into their constituent ions in an
between Brönsted-Lowry and Lewis concepts, aqueous medium, thereby acting as proton
as the base provides a lone pair in both the (H +) donors. Similarly, strong bases like
cases. However, in Lewis concept many lithium hydroxide (LiOH), sodium hydroxide
acids do not have proton. A typical example (NaOH), potassium hydroxide (KOH), caesium
is reaction of electron deficient species BF3 hydroxide (CsOH) and barium hydroxide
with NH3. Ba(OH)2 are almost completely dissociated
into ions in an aqueous medium giving
BF3 does not have a proton but still acts
hydroxyl ions, OH–. According to Arrhenius
as an acid and reacts with NH3 by accepting
concept they are strong acids and bases as
its lone pair of electrons. The reaction can be
they are able to completely dissociate and
represented by,
produce H3O+ and OH– ions respectively in
BF3 + :NH3 → BF3:NH3 the medium. Alternatively, the strength of an
Electron deficient species like AlCl3, Co3+, acid or base may also be gauged in terms of
Mg , etc. can act as Lewis acids while species
2+
Brönsted-Lowry concept of acids and bases,
like H2O, NH3, OH– etc. which can donate a wherein a strong acid means a good proton
pair of electrons, can act as Lewis bases. donor and a strong base implies a good proton
acceptor. Consider, the acid-base dissociation
Problem 6.15 equilibrium of a weak acid HA,
Classify the following species into Lewis HA(aq) + H2O(l)   H3O+(aq) + A–(aq)
acids and Lewis bases and show how these conjugate conjugate
act as such: acid base acid base
(a) HO – (b) F – (c) H+ (d) BCl3 In section 6.10.2 we saw that acid (or
Solution base) dissociation equilibrium is dynamic
involving a transfer of proton in forward and
(a) Hydroxyl ion is a Lewis base as it can reverse directions. Now, the question arises
donate an electron lone pair (:OH– ). that if the equilibrium is dynamic then with
(b) Flouride ion acts as a Lewis base passage of time which direction is favoured?
as it can donate any one of its four What is the driving force behind it? In order
electron lone pairs. to answer these questions we shall deal
(c) A proton is a Lewis acid as it can into the issue of comparing the strengths
accept a lone pair of electrons from of the two acids (or bases) involved in the
bases like hydroxyl ion and fluoride dissociation equilibrium. Consider the two
ion. acids HA and H 3O + present in the above
(d) BCl3 acts as a Lewis acid as it can mentioned acid-dissociation equilibrium.
accept a lone pair of electrons from We have to see which amongst them is a
species like ammonia or amine stronger proton donor. Whichever exceeds
molecules. in its tendency of donating a proton over the
other shall be termed as the stronger acid
6.11 IONIZATION OF ACIDS AND BASES and the equilibrium will shift in the direction
of weaker acid. Say, if HA is a stronger acid
Arrhenius concept of acids and bases
than H3O+, then HA will donate protons and
becomes useful in case of ionization of acids
not H3O+, and the solution will mainly contain
and bases as mostly ionizations in chemical
A– and H3O+ ions. The equilibrium moves in
and biological systems occur in aqueous
the direction of formation of weaker acid
medium. Strong acids like perchloric acid

Rationalised 2023-24

Unit 6.indd 192 9/12/2022 11:58:26 AM


EQUILIBRIUM 193

and weaker base because the stronger acid H2O(l) + H2O(l) H3O+(aq) + OH–(aq)
donates a proton to the stronger base. acid base conjugate conjugate
It follows that as a strong acid dissociates acid base
completely in water, the resulting base formed The dissociation constant is represented by,
would be very weak i.e., strong acids have K = [H3O+] [OH–] / [H2O] (6.26)
very weak conjugate bases. Strong acids
like perchloric acid (HClO4), hydrochloric acid The concentration of water is omitted from
(HCl), hydrobromic acid (HBr), hydroiodic acid the denominator as water is a pure liquid and
(HI), nitric acid (HNO3) and sulphuric acid its concentration remains constant. [H2O] is
(H2SO4) will give conjugate base ions ClO4–, Cl, incorporated within the equilibrium constant
Br–, I–, NO3– and HSO4– , which are much weaker to give a new constant, Kw, which is called the
bases than H2O. Similarly a very strong base ionic product of water.
would give a very weak conjugate acid. On the Kw = [H+][OH–] (6.27)
other hand, a weak acid say HA is only partially
The concentration of H has been found
+
dissociated in aqueous medium and thus, the
out experimentally as 1.0 × 10–7 M at 298 K.
solution mainly contains undissociated HA
And, as dissociation of water produces equal
molecules. Typical weak acids are nitrous
number of H+ and OH– ions, the concentration
acid (HNO2), hydrofluoric acid (HF) and acetic
of hydroxyl ions, [OH–] = [H+] = 1.0 × 10–7 M.
acid (CH3COOH). It should be noted that the
weak acids have very strong conjugate bases. Thus, the value of Kw at 298K,
For example, NH2–, O 2– and H – are very good Kw = [H3O+][OH–] = (1 × 10–7)2 = 1 × 10–14 M2
proton acceptors and thus, much stronger (6.28)
bases than H2O.
The value of Kw is temperature dependent
Certain water soluble organic compounds as it is an equilibrium constant.
like phenolphthalein and bromothymol blue
The density of pure water is 1000 g / L
behave as weak acids and exhibit different
and its molar mass is 18.0 g /mol. From this
colours in their acid (HIn) and conjugate base
the molarity of pure water can be given as,
(In– ) forms.
[H2O] = (1000 g /L)(1 mol/18.0 g) = 55.55 M.
HIn(aq) + H2O(l) H3O+(aq) + In–(aq)
Therefore, the ratio of dissociated water to
acid conjugate conjugate
that of undissociated water can be given as:
indicator acid base
10 –7 / (55.55) = 1.8 × 10–9 or ~ 2 in 10–9 (thus,
colour A colour B equilibrium lies mainly towards undissociated
Such compounds are useful as indicators water)
in acid-base titrations, and finding out H+ ion
concentration. We can distinguish acidic, neutral and
basic aqueous solutions by the relative values
6.11.1 The Ionization Constant of Water of the H3O+ and OH– concentrations:
and its Ionic Product
Acidic: [H3O+] > [OH– ]
Some substances like water are unique in
their ability of acting both as an acid and a Neutral: [H3O+] = [OH– ]
base. We have seen this in case of water in Basic : [H3O+] < [OH–]
section 6.10.2. In presence of an acid, HA it
accepts a proton and acts as the base while 6.11.2 The pH Scale
in the presence of a base, B– it acts as an Hydronium ion concentration in molarity is
acid by donating a proton. In pure water, one more conveniently expressed on a logarithmic
H2O molecule donates proton and acts as an scale known as the pH scale. The pH of a
acid and another water molecules accepts a solution is defined as the negative logarithm
proton and acts as a base at the same time.
The following equilibrium exists:
to base 10 of the activity a H   of hydrogen

Rationalised 2023-24

Unit 6.indd 193 9/12/2022 11:58:26 AM


194 chemistry

ion. In dilute solutions (< 0.01 M), activity when the hydrogen ion concentration, [H+]
of hydrogen ion (H+) is equal in magnitude changes by a factor of 100, the value of pH
to molarity represented by [H+]. It should changes by 2 units. Now you can realise why
be noted that activity has no units and is the change in pH with temperature is often
defined as: ignored.
= [H+] / mol L–1 Measurement of pH of a solution is very
From the definition of pH, the following essential as its value should be known
when dealing with biological and cosmetic
can be written,
applications. The pH of a solution can be
pH = – log aH+ = – log {[H+] / mol L–1} found roughly with the help of pH paper that
Thus, an acidic solution of HCl (10–2 M) has different colour in solutions of different
will have a pH = 2. Similarly, a basic solution pH. Now-a-days pH paper is available with
of NaOH having [OH–] =10–4 M and [H3O+] = four strips on it. The different strips have
10–10 M will have a pH = 10. At 25 °C, pure different colours (Fig. 6.11) at the same pH.
water has a concentration of hydrogen ions, The pH in the range of 1-14 can be determined
[H + ] = 10–7 M. Hence, the pH of pure water is with an accuracy of ~0.5 using pH paper.
given as:
pH = –log(10–7) = 7
Acidic solutions possess a concentration
of hydrogen ions, [H + ] > 10–7 M, while basic
solutions possess a concentration of hydrogen
ions, [H+] < 10–7 M. thus, we can summarise
that
Fig.6.11 pH-paper with four strips that may have
Acidic solution has pH < 7 different colours at the same pH
Basic solution has pH > 7
Neutral solution has pH = 7 For greater accuracy pH meters are used.
pH meter is a device that measures the
Now again, consider the equation (6.28)
pH-dependent electrical potential of the test
at 298 K
solution within 0.001 precision. pH meters
Kw = [H3O+] [OH–] = 10–14
of the size of a writing pen are now available
Taking negative logarithm on both sides in the market. The pH of some very common
of equation, we obtain substances are given in Table 6.5 (page 195).
–log Kw = – log {[H3O+] [OH–]}
Problem 6.16
= – log [H3O+] – log [OH–]
The concentration of hydrogen ion in a
= – log 10 –14
sample of soft drink is 3.8 × 10–3M. what
pKw = pH + pOH = 14 (6.29) is its pH ?
Note that although K w may change
with temperature the variations in pH with Solution
temperature are so small that we often pH = – log[3.8 × 10–3]
ignore it. = – {log[3.8] + log[10–3]}
pK w is a very important quantity for = – {(0.58) + (– 3.0)} = – { – 2.42} = 2.42
aqueous solutions and controls the relative Therefore, the pH of the soft drink is 2.42
concentrations of hydrogen and hydroxyl and it can be inferred that it is acidic.
ions as their product is a constant. It should
Problem 6.17
be noted that as the pH scale is logarithmic,
a change in pH by just one unit also means Calculate pH of a 1.0 × 10 –8 M solution of
change in [H+] by a factor of 10. Similarly, HCl.

Rationalised 2023-24

Unit 6.indd 194 9/12/2022 11:58:28 AM


EQUILIBRIUM 195

Table 6.5 The pH of Some Common Substances

Name of the Fluid pH Name of the Fluid pH

Saturated solution of NaOH ~15 Black Coffee 5.0


0.1 M NaOH solution 13 Tomato juice ~4.2
Lime water 10.5 Soft drinks and vinegar ~3.0
Milk of magnesia 10 Lemon juice ~2.2
Egg white, sea water 7.8 Gastric juice ~1.2
Human blood 7.4 1M HCl solution ~0
Milk 6.8 Concentrated HCl ~–1.0
Human Saliva 6.4

Solution equilibrium constant for the above discussed


acid-dissociation equilibrium:
2H2O (l) H3O+ (aq) + OH–(aq) Ka = c2α2 / c(1-α) = cα2 / 1-α
Kw = [OH ][H3O+]

Ka is called the dissociation or ionization
= 10–14 constant of acid HX. It can be represented
Let, x = [OH –] = [H 3O +] from H 2O. The alternatively in terms of molar concentration
H3O+ concentration is generated (i) from as follows,
the ionization of HCl dissolved i.e.,
Ka = [H+][X –] / [HX] (6.30)
HCl(aq) + H2O(l) H3O+ (aq) + Cl –(aq),
At a given temperature T, K a is a
and (ii) from ionization of H2O. In these very measure of the strength of the acid HX i.e.,
dilute solutions, both sources of H3O+ must larger the value of Ka, the stronger is the
be considered:
acid. Ka is a dimensionless quantity with
[H3O+] = 10–8 + x the understanding that the standard state
Kw = (10–8 + x)(x) = 10–14 concentration of all species is 1M.
or x2 + 10–8 x – 10–14 = 0 The values of the ionization constants
[OH – ] = x = 9.5 × 10–8 of some selected weak acids are given in
So, pOH = 7.02 and pH = 6.98 Table 6.6.
Table 6.6 The Ionization Constants of Some
6.11.3 Ionization Constants of Weak Acids Selected Weak Acids (at 298K)
Consider a weak acid HX that is partially Acid Ionization Constant,
ionized in the aqueous solution. The Ka
equilibrium can be expressed by:
Hydrofluoric Acid (HF) 3.5 × 10–4
HX(aq) + H2O(l) H3O+(aq) + X –(aq)
Nitrous Acid (HNO2) 4.5 × 10–4
Initial
Formic Acid (HCOOH) 1.8 × 10–4
concentration (M)
c 0 0 Niacin (C5H4NCOOH) 1.5 × 10–5
Let α be the extent of ionization Acetic Acid (CH3COOH) 1.74 × 10–5
Change (M) Benzoic Acid (C6H5COOH) 6.5 × 10–5
-cα +cα +cα Hypochlorous Acid (HCIO) 3.0 × 10–8
Equilibrium concentration (M) Hydrocyanic Acid (HCN) 4.9 × 10–10
c-cα cα cα Phenol (C6H5OH) 1.3 × 10–10
Here, c = initial concentration of the
undissociated acid, HX at time, t = 0. α = The pH scale for the hydrogen ion
extent up to which HX is ionized into ions. concentration has been so useful that besides
Using these notations, we can derive the pKw, it has been extended to other species and

Rationalised 2023-24

Unit 6.indd 195 11/2/2022 4:11:08 PM


196 chemistry

quantities. Thus, we have:


Solution
pKa = –log (Ka) (6.31)
The following proton transfer reactions are
Knowing the ionization constant, K a possible:
of an acid and its initial concentration, c, 1) HF + H2O H3O+ + F –
it is possible to calculate the equilibrium
Ka = 3.2 × 10–4
concentration of all species and also the
degree of ionization of the acid and the pH of 2) H2O + H2O H3O + OH –
+

the solution. Kw = 1.0 × 10–14


A general step-wise approach can be As Ka >> Kw, [1] is the principle reaction.
adopted to evaluate the pH of the weak HF + H2O H3O+ + F–
electrolyte as follows: Initial
Step 1. The species present before concentration (M)
dissociation are identified as Brönsted-Lowry 0.02 0 0
acids/bases. Change (M)
Step 2. Balanced equations for all possible –0.02α +0.02α +0.02α
reactions i.e., with a species acting both as
Equilibrium
acid as well as base are written.
Step 3. The reaction with the higher Ka is concentration (M)
identified as the primary reaction whilst the 0.02 – 0.02 α 0.02 α 0.02α
other is a subsidiary reaction. Substituting equilibrium concentrations
Step 4. Enlist in a tabular form the following in the equilibrium reaction for principal
values for each of the species in the primary reaction gives:
reaction Ka = (0.02α)2 / (0.02 – 0.02α)
(a) Initial concentration, c. = 0.02 α2 / (1 –α) = 3.2 × 10–4
(b) Change in concentration on proceeding We obtain the following quadratic equation:
to equilibrium in terms of α, degree of α2 + 1.6 × 10–2α – 1.6 × 10–2 = 0
ionization. The quadratic equation in α can be solved
(c) Equilibrium concentration. and the two values of the roots are:
Step 5. Substitute equilibrium concentrations α = + 0.12 and – 0.12
into equilibrium constant equation for The negative root is not acceptable and
principal reaction and solve for α. hence,
Step 6. Calculate the concentration of species α = 0.12
in principal reaction. This means that the degree of ionization,
Step 7. Calculate pH = – log[H3O+] α = 0.12, then equilibrium concentrations
of other species viz., HF, F – and H3O+ are
The above mentioned methodology has given by:
been elucidated in the following examples.
[H3O+] = [F –] = cα = 0.02 × 0.12
= 2.4 × 10–3 M
Problem 6.18 [HF] = c(1 – α) = 0.02 (1 – 0.12)
The ionization constant of HF is
3.2 × 10 –4 . Calculate the degree of = 17.6 × 10-3 M
dissociation of HF in its 0.02 M solution. pH = – log[H+] = –log(2.4 × 10–3) = 2.62
Calculate the concentration of all species Problem 6.19
present (H3O+, F – and HF) in the solution
and its pH. The pH of 0.1M monobasic acid is 4.50.
Calculate the concentration of species H+, A–

Rationalised 2023-24

Unit 6.indd 196 11/2/2022 4:11:08 PM


EQUILIBRIUM 197

and HA at equilibrium. Also, determine the Percent dissociation


value of Ka and pKa of the monobasic acid. = {[HOCl]dissociated / [HOCl]initial }× 100
Solution = 1.41 × 10–3 × 102/ 0.08 = 1.76 %.
pH = –log(1.41 × 10–3) = 2.85.
pH = – log [H+]
Therefore, [H+] = 10 –pH
= 10 –4.50 6.11.4 Ionization of Weak Bases
= 3.16 × 10 –5 The ionization of base MOH can be represented
by equation:
[H+] = [A–] = 3.16 × 10–5
MOH(aq) M+(aq) + OH–(aq)
Thus, Ka = [H+][A-] / [HA]
In a weak base there is partial ionization
[HA]eqlbm = 0.1 – (3.16 × 10-5)  0.1 of MOH into M+ and OH–, the case is similar
Ka = (3.16 × 10–5)2 / 0.1 = 1.0 × 10–8 to that of acid-dissociation equilibrium. The
equilibrium constant for base ionization
pKa = – log(10–8) = 8 is called base ionization constant and is
Alternatively, “Percent dissociation” is represented by Kb. It can be expressed in
another useful method for measure of terms of concentration in molarity of various
strength of a weak acid and is given as: species in equilibrium by the following
Percent dissociation equation:
= [HA]dissociated/[HA]initial × 100% (6.32) Kb = [M+][OH–] / [MOH] (6.33)
Alternatively, if c = initial concentration
Problem 6.20 of base and α = degree of ionization of base
Calculate the pH of 0.08M solution of i.e. the extent to which the base ionizes.
hypochlorous acid, HOCl. The ionization When equilibrium is reached, the equilibrium
constant of the acid is 2.5 × 10 –5. Determine constant can be written as:
the percent dissociation of HOCl. Kb = (cα)2 / c (1-α) = cα2 / (1-α)
Solution The values of the ionization constants of
HOCl(aq) + H2O (l) H3O+(aq) + ClO–(aq) some selected weak bases, Kb are given in
Table 6.7.
Initial concentration (M)
0.08 0 0 Table 6.7 The Values of the Ionization
Constant of Some Weak Bases at
Change to reach 298 K
equilibrium concentration Base Kb
(M)
Dimethylamine, (CH3)2NH 5.4 × 10–4
– x + x +x
Triethylamine, (C2H5)3N 6.45 × 10–5
equilibrium concentartion (M)
Ammonia, NH3 or NH4OH 1.77 × 10–5
0.08 – x x x
Quinine, (A plant product) 1.10 × 10–6
Ka = {[H3O ][ClO ] / [HOCl]}
+ –
Pyridine, C5H5N 1.77 × 10–9
= x / (0.08 –x)
2
Aniline, C6H5NH2 4.27 × 10–10
As x << 0.08, therefore 0.08 – x  0.08 Urea, CO (NH2)2 1.3 × 10–14
x2 / 0.08 = 2.5 × 10–5 Many organic compounds like amines
x2 = 2.0 × 10–6, thus, x = 1.41 × 10–3 are weak bases. Amines are derivatives of
[H+] = 1.41 × 10–3 M. ammonia in which one or more hydrogen
Therefore, atoms are replaced by another group. For
example, methylamine, codeine, quinine and

Rationalised 2023-24

Unit 6.indd 197 9/12/2022 11:58:28 AM


198 chemistry

nicotine all behave as very weak bases due to


Kb = 10–4.75 = 1.77 × 10–5 M
their very small Kb. Ammonia produces OH– in
aqueous solution: NH3 + H2O NH4+ + OH–
NH3(aq) + H2O(l) NH4+(aq) + OH–(aq) Initial concentration (M)
0.10 0.20 0
The pH scale for the hydrogen ion
concentration has been extended to get: Change to reach
pKb = –log (Kb) (6.34) equilibrium (M)
–x +x +x
At equilibrium (M)
Problem 6.21 0.10 – x 0.20 + x x
The pH of 0.004M hydrazine solution is 9.7. Kb = [NH4+][OH–] / [NH3]
Calculate its ionization constant Kb and pKb.
= (0.20 + x)(x) / (0.1 – x) = 1.77 × 10–5
Solution
As Kb is small, we can neglect x in comparison
NH2NH2 + H2O NH2NH3 +
+ OH
– to 0.1M and 0.2M. Thus,
From the pH we can calculate the hydrogen [OH–] = x = 0.88 × 10–5
ion concentration. Knowing hydrogen ion Therefore, [H+] = 1.12 × 10–9
concentration and the ionic product of
pH = – log[H+] = 8.95.
water we can calculate the concentration
of hydroxyl ions. Thus we have:
6.11.5 Relation between Ka and Kb
[H+] = antilog (–pH) As seen earlier in this chapter, Ka and Kb
= antilog (–9.7) = 1.67 ×10–10 represent the strength of an acid and a base,
[OH–] = Kw / [H+] = 1 × 10–14 / 1.67 × 10–10 respectively. In case of a conjugate acid-base
= 5.98 × 10–5 pair, they are related in a simple manner so
that if one is known, the other can be deduced.
The concentration of the corresponding
Considering the example of NH4+ and NH3
hydrazinium ion is also the same as that
of hydroxyl ion. The concentration of both we see,
these ions is very small so the concentration NH4+(aq) + H2O(l) H3O+(aq) + NH3(aq)
of the undissociated base can be taken Ka = [H3O+][ NH3] / [NH4+] = 5.6 × 10–10
equal to 0.004M.
NH3(aq) + H2O(l) NH4+(aq) + OH–(aq)
Thus,
Kb =[ NH4 ][ OH ] / NH3 = 1.8 × 10–5
+ –

Kb = [NH2NH3+][OH–] / [NH2NH2]
Net: 2 H2O(l) H3O+(aq) + OH –(aq)
= (5.98 × 10–5)2 / 0.004 = 8.96 × 10–7
Kw = [H3O+][ OH– ] = 1.0 × 10–14 M
pKb = –logKb = –log(8.96 × 10–7) = 6.04.
Where, Ka represents the strength of NH4+
Problem 6.22 as an acid and Kb represents the strength of
Calculate the pH of the solution in which NH3 as a base.
0.2M NH4Cl and 0.1M NH3 are present. The It can be seen from the net reaction that
pKb of ammonia solution is 4.75. the equilibrium constant is equal to the
product of equilibrium constants Ka and Kb
Solution
for the reactions added. Thus,
NH3 + H2O NH4+ + OH– Ka × K b = {[H3O+][ NH3] / [NH4+ ]} × {[NH4+]
The ionization constant of NH3, [OH–] / [NH3]}
Kb = antilog (–pKb) i.e. = [H3O+][OH–] = Kw
= (5.6 ×10–10) × (1.8 × 10–5) = 1.0 × 10–14 M

Rationalised 2023-24

Unit 6.indd 198 9/12/2022 11:58:28 AM


EQUILIBRIUM 199

This can be extended to make a


NH3 + H2O NH4+ + OH–
generalisation. The equilibrium constant
for a net reaction obtained after adding We use equation (6.33) to calculate
hydroxyl ion concentration,
two (or more) reactions equals the product
of the equilibrium constants for individual [OH–] = c α = 0.05 α
reactions: Kb = 0.05 α2 / (1 – α)
K NET = K1 × K2 × …… (6.35) The value of α is small, therefore the
quadratic equation can be simplified by
Similarly, in case of a conjugate acid-base neglecting α in comparison to 1 in the
pair, denominator on right hand side of the
Ka × Kb = Kw (6.36) equation,
Knowing one, the other can be obtained. Thus,
It should be noted that a strong acid will have Kb = c α2 or α = √ (1.77 × 10–5 / 0.05)
a weak conjugate base and vice-versa. = 0.018.
Alternatively, the above expression [OH ] = c α = 0.05 × 0.018 = 9.4 × 10–4M.

K w = K a × K b , can also be obtained by


considering the base-dissociation equilibrium [H+] = Kw / [OH–] = 10–14 / (9.4 × 10–4)
reaction: = 1.06 × 10–11
B(aq) + H2O(l) BH+(aq) + OH–(aq) pH = –log(1.06 × 10–11) = 10.97.
K b = [BH+][OH–] / [B] Now, using the relation for conjugate
As the concentration of water remains acid-base pair,
constant it has been omitted from the Ka × Kb = Kw
denominator and incorporated within the
using the value of K b of NH 3 from
dissociation constant. Then multiplying and
Table 6.7.
dividing the above expression by [H+], we get:
Kb = [BH+][OH –][H+] / [B][H+] We can determine the concentration of
conjugate acid NH4+
={[ OH–][H+]}{[BH+] / [B][H+]}
Ka = Kw / Kb = 10–14 / 1.77 × 10–5
= Kw / Ka
or Ka × Kb = Kw = 5.64 × 10–10.
It may be noted that if we take negative
logarithm of both sides of the equation, then 6.11.6 Di- and Polybasic Acids and Di-
pK values of the conjugate acid and base are and Polyacidic Bases
related to each other by the equation: Some of the acids like oxalic acid, sulphuric
pKa + pKb = pKw = 14 (at 298K) acid and phosphoric acids have more than
one ionizable proton per molecule of the
Problem 6.23 acid. Such acids are known as polybasic or
polyprotic acids.
Determine the degree of ionization and pH of
The ionization reactions for example for
a 0.05M of ammonia solution. The ionization
a dibasic acid H2X are represented by the
constant of ammonia can be taken from
equations:
Table 6.7. Also, calculate the ionization
H2X(aq) H+(aq) + HX–(aq)
constant of the conjugate acid of ammonia.
HX–(aq) H+(aq) + X2–(aq)
Solution
And the corresponding equilibrium
The ionization of NH3 in water is represented constants are given below:
by equation:
Ka1= {[H+][HX–]} / [H2X] and

Rationalised 2023-24

Unit 6.indd 199 9/12/2022 11:58:28 AM


200 chemistry

Ka2 = {[H+][X2-]} / [HX-] In general, when strength of H-A bond


decreases, that is, the energy required to break
Here, Ka1and Ka2are called the first and second the bond decreases, HA becomes a stronger
ionization constants respectively of the acid H2 acid. Also, when the H-A bond becomes more
X. Similarly, for tribasic acids like H3PO4 we polar i.e., the electronegativity difference
have three ionization constants. The values between the atoms H and A increases and
of the ionization constants for some common there is marked charge separation, cleavage
polyprotic acids are given in Table 6.8. of the bond becomes easier thereby increasing
Table 6.8 The Ionization Constants of Some the acidity.
Common Polyprotic Acids (298K)
But it should be noted that while comparing
elements in the same group of the periodic
table, H-A bond strength is a more important
factor in determining acidity than its polar
nature. As the size of A increases down the
group, H-A bond strength decreases and so
the acid strength increases. For example,
Size increases
HF << HCl << HBr << HI
It can be seen that higher order ionization
constants (K a2, K a3) are smaller than the Acid strength increases
Similarly, H2S is stronger acid than H2O.
lower order ionization constant (K a1) of a
polyprotic acid. The reason for this is that But, when we discuss elements in the same
it is more difficult to remove a positively row of the periodic table, H-A bond polarity
charged proton from a negative ion due to becomes the deciding factor for determining
electrostatic forces. This can be seen in the the acid strength. As the electronegativity
case of removing a proton from the uncharged of A increases, the strength of the acid also
H2CO3 as compared from a negatively charged increases. For example,
HCO3–. Similarly, it is more difficult to remove
Electronegativity of A increases
a proton from a doubly charged HPO42– anion
as compared to H2PO4–. CH4 < NH3 < H2O < HF
Polyprotic acid solutions contain a Acid strength increases
mixture of acids like H2A, HA– and A2– in case
of a diprotic acid. H2A being a strong acid, the 6.11.8 Common Ion Effect in the
primary reaction involves the dissociation of Ionization of Acids and Bases
H2 A, and H3O+ in the solution comes mainly Consider an example of acetic acid dissociation
from the first dissociation step. equilibrium represented as:
CH3COOH(aq) H+(aq) + CH3COO– (aq)
6.11.7 Factors Affecting Acid Strength
Having discussed quantitatively the strengths or HAc(aq) H+ (aq) + Ac– (aq)
of acids and bases, we come to a stage where Ka = [H+][Ac– ] / [HAc]
we can calculate the pH of a given acid
Addition of acetate ions to an acetic
solution. But, the curiosity rises about why
acid solution results in decreasing the
should some acids be stronger than others?
concentration of hydrogen ions, [H+]. Also,
What factors are responsible for making
them stronger? The answer lies in its being a if H+ ions are added from an external source
complex phenomenon. But, broadly speaking then the equilibrium moves in the direction
we can say that the extent of dissociation of of undissociated acetic acid i.e., in a direction
an acid depends on the strength and polarity of reducing the concentration of hydrogen
of the H-A bond. ions, [H+]. This phenomenon is an example

Rationalised 2023-24

Unit 6.indd 200 9/12/2022 11:58:28 AM


EQUILIBRIUM 201

of common ion effect. It can be defined as


Thus, x = 1.33 × 10–3 = [OH–]
a shift in equilibrium on adding a substance
that provides more of an ionic species already Therefore, [H+] = Kw / [OH–] = 10–14 /
present in the dissociation equilibrium. (1.33 × 10–3) = 7.51 × 10–12
Thus, we can say that common ion effect is
a phenomenon based on the Le Chatelier’s pH = –log (7.5 × 10–12) = 11.12
principle discussed in section 6.8. On addition of 25 mL of 0.1M HCl
In order to evaluate the pH of the solution solution (i.e., 2.5 mmol of HCl) to 50
resulting on addition of 0.05M acetate ion to mL of 0.1M ammonia solution (i.e., 5
mmol of NH3), 2.5 mmol of ammonia
0.05M acetic acid solution, we shall consider
molecules are neutralized. The resulting
the acetic acid dissociation equilibrium once
75 mL solution contains the remaining
again, unneutralized 2.5 mmol of NH3 molecules
HAc(aq) H+(aq) + Ac–(aq) and 2.5 mmol of NH4+.
Initial concentration (M) NH3 + HCl → NH4+ + Cl–
0.05 0 0.05 2.5 2.5 0 0
Let x be the extent of ionization of acetic At equilibrium
acid. 0 0 2.5 2.5
Change in concentration (M) The resulting 75 mL of solution contains
2.5 mmol of NH4+ ions (i.e., 0.033 M) and
–x +x +x
2.5 mmol (i.e., 0.033 M ) of uneutralised
Equilibrium concentration (M) NH3 molecules. This NH3 exists in the
0.05-x x 0.05+x following equilibrium:
Therefore, NH4OH   NH4+ + OH–
Ka= [H+][Ac– ]/[H Ac] = {(0.05+x)(x)}/(0.05-x) 0.033M – y y y
where, y = [OH ] = [NH4 ]
– +
As Ka is small for a very weak acid, x<<0.05.
Hence, (0.05 + x) ≈ (0.05 – x) ≈ 0.05 The final 75 mL solution after
neutralisation already contains
Thus, 2.5 m mol NH4+ ions (i.e. 0.033M), thus
1.8 × 10–5 = (x) (0.05 + x) / (0.05 – x) total concentration of NH4+ ions is given
= x(0.05) / (0.05) = x = [H+] = 1.8 × 10–5M as:
[NH4+] = 0.033 + y
pH = – log(1.8 × 10–5) = 4.74
As y is small, [NH4OH]  0.033 M and
Problem 6.24 [NH4+]  0.033M.
We know,
Calculate the pH of a 0.10M ammonia
solution. Calculate the pH after 50.0 mL Kb = [NH4+][OH–] / [NH4OH]
of this solution is treated with 25.0 mL of = y (0.033)/(0.033) = 1.77 × 10–5 M
0.10M HCl. The dissociation constant of
ammonia, Kb = 1.77 × 10–5 Thus, y = 1.77 × 10–5 = [OH–]

Solution [H+] = 10–14 / 1.77 × 10–5 = 0.56 × 10–9


Hence, pH = 9.24
NH3 + H2O → NH4+ + OH–
Kb = [NH4+][OH–] / [NH3] = 1.77 × 10–5
Before neutralization, 6.11.9 Hydrolysis of Salts and the pH of
their Solutions
[NH4+] = [OH–] = x
[NH3] = 0.10 – x  0.10 Salts formed by the reactions between acids
and bases in definite proportions, undergo
x2 / 0.10 = 1.77 × 10–5
ionization in water. The cations/anions

Rationalised 2023-24

Unit 6.indd 201 9/12/2022 11:58:29 AM


202 chemistry

formed on ionization of salts either exist as increased of H+ ion concentration in solution


hydrated ions in aqueous solutions or interact making the solution acidic. Thus, the pH of
with water to reform corresponding acids/ NH4Cl solution in water is less than 7.
bases depending upon the nature of salts. Consider the hydrolysis of CH3COONH4
The later process of interaction between salt formed from weak acid and weak base.
water and cations/anions or both of salts The ions formed undergo hydrolysis as follow:
is called hydrolysis. The pH of the solution
gets affected by this interaction. The cations CH3COO– + NH4+ + H2O CH3COOH +
(e.g., Na+, K+, Ca2+, Ba2+, etc.) of strong bases NH4OH
and anions (e.g., Cl–, Br–, NO3–, ClO4– etc.) of CH3COOH and NH4OH, also remain into
strong acids simply get hydrated but do not partially dissociated form:
hydrolyse, and therefore the solutions of CH3COOH CH3COO– + H+
salts formed from strong acids and bases are
NH4OH NH4+ + OH–
neutral i.e., their pH is 7. However, the other
category of salts do undergo hydrolysis. H2O H+ + OH–
We now consider the hydrolysis of the Without going into detailed calculation,
salts of the following types : it can be said that degree of hydrolysis is
(i) salts of weak acid and strong base e.g., independent of concentration of solution, and
CH3COONa. pH of such solutions is determined by their
(ii) salts of strong acid and weak base e.g., pK values:
NH4Cl, and pH = 7 + ½ (pKa – pKb) (6.38)
(iii) salts of weak acid and weak base, e.g., The pH of solution can be greater than 7,
CH3COONH4. if the difference is positive and it will be less
In the first case, CH3COONa being a salt of than 7, if the difference is negative.
weak acid, CH3COOH and strong base, NaOH
Problem 6.25
gets completely ionised in aqueous solution.
The pKa of acetic acid and pKb of ammonium
CH3COONa(aq) → CH3COO– (aq)+ Na+(aq)
hydroxide are 4.76 and 4.75 respectively.
Acetate ion thus formed undergoes Calculate the pH of ammonium acetate
hydrolysis in water to give acetic acid and solution.
OH– ions
Solution
CH3COO–(aq)+H2O(l) CH3COOH(aq)+OH–(aq)
pH = 7 + ½ [pKa – pKb]
Acetic acid being a weak acid
= 7 + ½ [4.76 – 4.75]
(Ka = 1.8 × 10–5) remains mainly unionised in
solution. This results in increase of OH– ion = 7 + ½ [0.01] = 7 + 0.005 = 7.005
concentration in solution making it alkaline.
The pH of such a solution is more than 7. 6.12 BUFFER SOLUTIONS
Similarly, NH 4 Cl formed from weak Many body fluids e.g., blood or urine have
base, NH4OH and strong acid, HCl, in water definite pH and any deviation in their pH
dissociates completely. indicates malfunctioning of the body. The
+ control of pH is also very important in
NH4Cl(aq) → NH 4(aq) +Cl– (aq) many chemical and biochemical processes.
Ammonium ions undergo hydrolysis with Many medical and cosmetic formulations
water to form NH4OH and H+ ions require that these be kept and administered
+
NH 4 (aq) + H2O (1) NH4OH(aq) + H+(aq) at a particular pH. The solutions which
Ammonium hydroxide is a weak base resist change in pH on dilution or with
(K b = 1.77 × 10 –5) and therefore remains the addition of small amounts of acid or
almost unionised in solution. This results in alkali are called Buffer Solutions. Buffer

Rationalised 2023-24

Unit 6.indd 202 9/12/2022 11:58:29 AM


EQUILIBRIUM 203

solutions of known pH can be prepared from acid present in the mixture. Since acid is a
the knowledge of pKa of the acid or pKb of base weak acid, it ionises to a very little extent and
and by controlling the ratio of the salt and acid concentration of [HA] is negligibly different
or salt and base. A mixture of acetic acid and from concentration of acid taken to form
sodium acetate acts as buffer solution around buffer. Also, most of the conjugate base, [A—],
pH 4.75 and a mixture of ammonium chloride comes from the ionisation of salt of the acid.
and ammonium hydroxide acts as a buffer Therefore, the concentration of conjugate
around pH 9.25. You will learn more about base will be negligibly different from the
buffer solutions in higher classes. concentration of salt. Thus, equation (6.40)
takes the form:
6.12.1 Designing Buffer Solution
Knowledge of pK a , pK b and equilibrium [Salt]
pH=pK a + log
constant help us to prepare the buffer solution [Acid]
of known pH. Let us see how we can do this. In the equation (6.39), if the concentration
Preparation of Acidic Buffer of [A—] is equal to the concentration of [HA],
To prepare a buffer of acidic pH we use weak then pH = pKa because value of log 1 is zero.
acid and its salt formed with strong base. Thus if we take molar concentration of acid
and salt (conjugate base) same, the pH of the
We develop the equation relating the pH, the
buffer solution will be equal to the pKa of the
equilibrium constant, Ka of weak acid and
acid. So for preparing the buffer solution of
ratio of concentration of weak acid and its
the required pH we select that acid whose pKa
conjugate base. For the general case where
is close to the required pH. For acetic acid
the weak acid HA ionises in water,
pKa value is 4.76, therefore pH of the buffer
HA + H2O  H3O+ + A– solution formed by acetic acid and sodium
For which we can write the expression acetate taken in equal molar concentration
will be around 4.76.
A similar analysis of a buffer made with
a weak base and its conjugate acid leads to
Rearranging the expression we have,
the result,
[Conjugate acid,BH+ ]
pOH= pK b +log
[Base,B]
Taking logarithm on both the sides and (6.41)
rearranging the terms we get — pH of the buffer solution can be calculated
by using the equation pH + pOH =14.
We know that pH + pOH = pK w and
Or pKa + pKb = pKw. On putting these values in
equation (6.41) it takes the form as follows:
(6.39) [Conjugate acid,BH ]
pK w - pH = pK w  pK a  log
[Base,B]
or
[Conjugate acid,BH+ ]
pH = pK a + log
(6.40) [Base,B]
(6.42)
The expression (6.40) is known as If molar concentration of base and its
Henderson–Hasselbalch equation. The conjugate acid (cation) is same then pH of
the buffer solution will be same as pKa for the
quantity is the ratio of concentration base. pKa value for ammonia is s9.25; therefore
a buffer of pH close to 9.25 can be obtained
of conjugate base (anion) of the acid and the by taking ammonia solution and ammonium

Rationalised 2023-24

Unit 6.indd 203 9/12/2022 11:58:29 AM


204 chemistry

chloride solution of same molar concentration. We shall now consider the equilibrium
For a buffer solution formed by ammonium between the sparingly soluble ionic salt and
chloride and ammonium hydroxide, equation its saturated aqueous solution.
(6.42) becomes:
6.13.1 Solubility Product Constant
[Conjugate acid,BH+ ]
pH = 9.25 + log Let us now have a solid like barium sulphate
[Base,B] in contact with its saturated aqueous solution.
pH of the buffer solution is not affected by The equilibrium between the undisolved solid
dilution because ratio under the logarithmic and the ions in a saturated solution can be
term remains unchanged. represented by the equation:

6.13 SOLUBILITY EQUILIBRIA OF BaSO4(s)  Ba2+(aq) + SO42–(aq),


SPARINGLY SOLUBLE SALTS
We have already known that the solubility of The equilibrium constant is given by the
ionic solids in water varies a great deal. Some of equation:
these (like calcium chloride) are so soluble that K = {[Ba2+][SO42–]} / [BaSO4]
they are hygroscopic in nature and even absorb
For a pure solid substance the
water vapour from atmosphere. Others (such
concentration remains constant and we can
as lithium fluoride) have so little solubility
write
that they are commonly termed as insoluble.
The solubility depends on a number of factors Ksp = K[BaSO4] = [Ba2+][SO42–] (6.43)
important amongst which are the lattice We call Ksp the solubility product constant
enthalpy of the salt and the solvation enthalpy or simply solubility product. The experimental
of the ions in a solution. For a salt to dissolve value of Ksp in above equation at 298K is
in a solvent the strong forces of attraction 1.1 × 10–10. This means that for solid barium
between its ions (lattice enthalpy) must be sulphate in equilibrium with its saturated
overcome by the ion-solvent interactions. The solution, the product of the concentrations
solvation enthalpy of ions is referred to in of barium and sulphate ions is equal
terms of solvation which is always negative i.e. to its solubility product constant. The
energy is released in the process of solvation. concentrations of the two ions will be equal to
The amount of solvation enthalpy depends on the molar solubility of the barium sulphate.
the nature of the solvent. In case of a non- If molar solubility is S, then
polar (covalent) solvent, solvation enthalpy is
1.1 × 10–10 = (S)(S) = S2
small and hence, not sufficient to overcome
lattice enthalpy of the salt. Consequently, the or S = 1.05 × 10–5.
salt does not dissolve in non-polar solvent. As Thus, molar solubility of barium sulphate
a general rule, for a salt to be able to dissolve will be equal to 1.05 × 10–5 mol L–1.
in a particular solvent its solvation enthalpy A salt may give on dissociation two or
must be greater than its lattice enthalpy so more than two anions and cations carrying
that the latter may be overcome by former. different charges. For example, consider a salt
Each salt has its characteristic solubility which like zirconium phosphate of molecular formula
depends on temperature. We classify salts on (Zr4+)3(PO43–)4. It dissociates into 3 zirconium
the basis of their solubility in the following cations of charge +4 and 4 phosphate
three categories. anions of charge –3. If the molar solubility of
zirconium phosphate is S, then it can be seen
Category I Soluble Solubility > 0.1M from the stoichiometry of the compound that
Category II Slightly 0.01M<Solubility< 0.1M
[Zr4+] = 3S and [PO43–] = 4S
Soluble
Category III Sparingly Solubility < 0.01M and Ksp = (3S)3 (4S)4 = 6912 (S)7
Soluble or S = {Ksp / (33 × 44)}1/7 = (Ksp / 6912)1/7

Rationalised 2023-24

Unit 6.indd 204 9/12/2022 11:58:30 AM


EQUILIBRIUM 205

p q
A solid salt of the general formula M x X y Table 6.9 The Solubility Product Constants,
with molar solubility S in equilibrium with Ksp of Some Common Ionic Salts at
its saturated solution may be represented by 298K.
the equation:
MxXy(s) xMp+(aq) + yXq– (aq)
(where x × p = y × q–)
+

And its solubility product constant is


given by:
Ksp = [Mp+]x[Xq– ]y = (xS)x(yS)y (6.44)
= xx . yy . S(x + y)
S(x + y) = Ksp / xx . yy
S = (Ksp / xx . yy)1 / x + y (6.45)
The term Ksp in equation is given by Qsp
(section 6.6.2) when the concentration of one
or more species is not the concentration under
equilibrium. Obviously under equilibrium
conditions Ksp = Qsp but otherwise it gives the
direction of the processes of precipitation or
dissolution. The solubility product constants
of a number of common salts at 298K are
given in Table 6.9.

Problem 6.26
Calculate the solubility of A2 X3 in pure
water, assuming that neither kind of ion
reacts with water. The solubility product of
A2X3, Ksp = 1.1 × 10–23.
Solution
A2X3 → 2A3+ + 3X2–
Ksp = [A3+]2 [X2–]3 = 1.1 × 10–23
If S = solubility of A2X3, then
[A3+] = 2S; [X2–] = 3S
therefore, Ksp = (2S)2(3S)3 = 108S5
  = 1.1 × 10–23
thus, S5 = 1 × 10–25
S = 1.0 × 10–5 mol/L.
Problem 6.27
The values of Ksp of two sparingly soluble
salts Ni(OH)2 and AgCN are 2.0 × 10–15 and
6 × 0–17 respectively. Which salt is more
soluble? Explain.
Solution
AgCN Ag+ + CN–

Rationalised 2023-24

Unit 6.indd 205 9/12/2022 11:58:30 AM


206 chemistry

Ksp = [Ag+][CN–] = 6 × 10–17 Dissolution of S mol/L of Ni(OH)2 provides


Ni(OH)2 Ni2+ + 2OH– S mol/L of Ni2+ and 2S mol/L of OH–, but
the total concentration of OH– = (0.10 + 2S)
Ksp = [Ni ][OH–]2 = 2 × 10–15
2+
mol/L because the solution already contains
Let [Ag+] = S1, then [CN-] = S1 0.10 mol/L of OH– from NaOH.
Let [Ni2+] = S2, then [OH–] = 2S2
Ksp = 2.0 × 10–15 = [Ni2+] [OH–]2
S12 = 6 × 10–17 , S1 = 7.8 × 10–9
= (S) (0.10 + 2S)2
(S2)(2S2)2 = 2 × 10–15, S2 = 0.58 × 10–4
As Ksp is small, 2S << 0.10,
Ni(OH)2 is more soluble than AgCN.
thus, (0.10 + 2S) ≈ 0.10
6.13.2 Common Ion Effect on Solubility Hence,
of Ionic Salts 2.0 × 10–15 = S (0.10)2
It is expected from Le Chatelier’s principle S = 2.0 × 10–13 M = [Ni2+]
that if we increase the concentration of any
one of the ions, it should combine with the
ion of its opposite charge and some of the The solubility of salts of weak acids like
salt will be precipitated till once again Ksp = phosphates increases at lower pH. This is
Qsp. Similarly, if the concentration of one of because at lower pH the concentration of the
the ions is decreased, more salt will dissolve anion decreases due to its protonation. This
to increase the concentration of both the ions in turn increase the solubility of the salt so
till once again Ksp = Qsp. This is applicable that Ksp = Qsp. We have to satisfy two equilibria
even to soluble salts like sodium chloride simultaneously i.e.,
except that due to higher concentrations
of the ions, we use their activities instead Ksp = [M+] [X–],
of their molarities in the expression for
Qsp. Thus if we take a saturated solution of
sodium chloride and pass HCl gas through it,
then sodium chloride is precipitated due to
increased concentration (activity) of chloride
ion available from the dissociation of HCl. [X –] / [HX] = Ka/[H+]
Sodium chloride thus obtained is of very Taking inverse of both side and adding 1
high purity and we can get rid of impurities we get
like sodium and magnesium sulphates. The
common ion effect is also used for almost HX   1  H  
1
complete precipitation of a particular ion  X   Ka
as its sparingly soluble salt, with very low
HX   H   H   K a
value of solubility product for gravimetric   
estimation. Thus we can precipitate silver ion  X   Ka
as silver chloride, ferric ion as its hydroxide
(or hydrated ferric oxide) and barium ion as Now, again taking inverse, we get
its sulphate for quantitative estimations. [X–] / {[X–] + [HX]} = f = Ka/(Ka + [H+]) and it
can be seen that ‘f’ decreases as pH decreases.
Problem 6.28 If S is the solubility of the salt at a given
Calculate the molar solubility of Ni(OH)2 in pH then
0.10 M NaOH. The ionic product of Ni(OH)2
is 2.0 × 10–15. Ksp = [S] [f S] = S2 {Ka/(Ka + [H+])} and
S = {Ksp ([H+] + Ka)/Ka}1/2  (6.46)
Solution
Thus solubility S increases with increase
Let the solubility of Ni(OH)2 be equal to S. in [H+] or decrease in pH.

Rationalised 2023-24

Unit 6.indd 206 9/12/2022 11:58:30 AM


EQUILIBRIUM 207

SUMMARY

When the number of molecules leaving the liquid to vapour equals the number of molecules
returning to the liquid from vapour, equilibrium is said to be attained and is dynamic in
nature. Equilibrium can be established for both physical and chemical processes and at
this stage rate of forward and reverse reactions are equal. Equilibrium constant, Kc is
expressed as the concentration of products divided by reactants, each term raised to the
stoichiometric coefficient.

For reaction, a A + b B c C +d D
Kc = [C]c[D]d/[A]a[B]b
Equilibrium constant has constant value at a fixed temperature and at this stage all the
macroscopic properties such as concentration, pressure, etc. become constant. For a gaseous
reaction equilibrium constant is expressed as Kp and is written by replacing concentration
terms by partial pressures in Kc expression. The direction of reaction can be predicted by
reaction quotient Qc which is equal to Kc at equilibrium. Le Chatelier’s principle states that
the change in any factor such as temperature, pressure, concentration, etc. will cause the
equilibrium to shift in such a direction so as to reduce or counteract the effect of the change.
It can be used to study the effect of various factors such as temperature, concentration,
pressure, catalyst and inert gases on the direction of equilibrium and to control the yield of
products by controlling these factors. Catalyst does not effect the equilibrium composition
of a reaction mixture but increases the rate of chemical reaction by making available a new
lower energy pathway for conversion of reactants to products and vice-versa.

All substances that conduct electricity in aqueous solutions are called electrolytes. Acids,
bases and salts are electrolytes and the conduction of electricity by their aqueous solutions
is due to anions and cations produced by the dissociation or ionization of electrolytes
in aqueous solution. The strong electrolytes are completely dissociated. In weak electrolytes
there is equilibrium between the ions and the unionized electrolyte molecules. According to
Arrhenius, acids give hydrogen ions while bases produce hydroxyl ions in their aqueous
solutions. Brönsted-Lowry on the other hand, defined an acid as a proton donor and a base as
a proton acceptor. When a Brönsted-Lowry acid reacts with a base, it produces its conjugate
base and a conjugate acid corresponding to the base with which it reacts. Thus a conjugate
pair of acid-base differs only by one proton. Lewis further generalised the definition of an
acid as an electron pair acceptor and a base as an electron pair donor. The expressions
for ionization (equilibrium) constants of weak acids (Ka) and weak bases (Kb) are developed
using Arrhenius definition. The degree of ionization and its dependence on concentration and
common ion are discussed. The pH scale (pH = –log[H+]) for the hydrogen ion concentration
(activity) has been introduced and extended to other quantities (pOH = – log[OH–]); pKa = –log[Ka];
pKb = –log[Kb]; and pKw = –log[Kw] etc.). The ionization of water has been considered and we
note that the equation: pH + pOH = pKw is always satisfied. The salts of strong acid and
weak base, weak acid and strong base, and weak acid and weak base undergo hydrolysis in
aqueous solution. The definition of buffer solutions, and their importance are discussed
briefly. The solubility equilibrium of sparingly soluble salts is discussed and the equilibrium
constant is introduced as solubility product constant (Ksp). Its relationship with solubility
of the salt is established. The conditions of precipitation of the salt from their solutions
or their dissolution in water are worked out. The role of common ion and the solubility of
sparingly soluble salts is also discussed.

Rationalised 2023-24

Unit 6.indd 207 9/12/2022 11:58:31 AM


208 chemistry

SUGGESTED ACTIVITIES FOR STUDENTS REGARDING THIS UNIT


(a) The student may use pH paper in determining the pH of fresh juices of various vegetables
and fruits, soft drinks, body fluids and also that of water samples available.
(b) The pH paper may also be used to determine the pH of different salt solutions and from
that he/she may determine if these are formed from strong/weak acids and bases.
(c) They may prepare some buffer solutions by mixing the solutions of sodium acetate and
acetic acid and determine their pH using pH paper.
(d) They may be provided with different indicators to observe their colours in solutions of
varying pH.
(e) They may perform some acid-base titrations using indicators.
(f) They may observe common ion effect on the solubility of sparingly soluble salts.
(g) If pH meter is available in their school, they may measure the pH with it and compare
the results obtained with that of the pH paper.

EXERCISES

6.1 A liquid is in equilibrium with its vapour in a sealed container at a fixed temperature.
The volume of the container is suddenly increased.

a) What is the initial effect of the change on vapour pressure?

b) How do rates of evaporation and condensation change initially?

c) What happens when equilibrium is restored finally and what will be the final vapour
pressure?

6.2 What is Kc for the following equilibrium when the equilibrium concentration of each
substance is: [SO2]= 0.60M, [O2] = 0.82M and [SO3] = 1.90M ?

2SO2(g) + O2(g) 2SO3(g)


6.3 At a certain temperature and total pressure of 105Pa, iodine vapour contains 40% by
volume of I atoms

I2 (g) 2I (g)
Calculate Kp for the equilibrium.

6.4 Write the expression for the equilibrium constant, Kc for each of the following reactions:

(i) 2NOCl (g) 2NO (g) + Cl2 (g)


(ii) 2Cu(NO3)2 (s) 2CuO (s) + 4NO2 (g) + O2 (g)
(iii) CH3COOC2H5(aq) + H2O(l) CH3COOH (aq) + C2H5OH (aq)
(iv) Fe3+ (aq) + 3OH– (aq) Fe(OH)3 (s)
(v) I2 (s) + 5F2 2IF5
6.5 Find out the value of Kc for each of the following equilibria from the value of Kp:

(i) 2NOCl (g) 2NO (g) + Cl2 (g); Kp= 1.8 × 10–2 at 500 K
(ii) CaCO3 (s) CaO(s) + CO2(g); Kp= 167 at 1073 K

Rationalised 2023-24

Unit 6.indd 208 9/12/2022 11:58:31 AM


EQUILIBRIUM 209

6.6 For the following equilibrium, Kc= 6.3 × 1014 at 1000 K


NO (g) + O3 (g) NO2 (g) + O2 (g)
Both the forward and reverse reactions in the equilibrium are elementary bimolecular
reactions. What is Kc, for the reverse reaction?
6.7 Explain why pure liquids and solids can be ignored while writing the equilibrium
constant expression?

6.8 Reaction between N2 and O2– takes place as follows:


2N2 (g) + O2 (g) 2N2O (g)
If a mixture of 0.482 mol N2 and 0.933 mol of O2 is placed in a 10 L reaction vessel
and allowed to form N2O at a temperature for which Kc= 2.0 × 10–37, determine the
composition of equilibrium mixture.

6.9 Nitric oxide reacts with Br2 and gives nitrosyl bromide as per reaction given below:
2NO (g) + Br2 (g) 2NOBr (g)
When 0.087 mol of NO and 0.0437 mol of Br2 are mixed in a closed container at
constant temperature, 0.0518 mol of NOBr is obtained at equilibrium. Calculate
equilibrium amount of NO and Br2 .

6.10 At 450K, Kp= 2.0 × 1010/bar for the given reaction at equilibrium.
2SO2(g) + O2(g) 2SO3 (g)
What is Kc at this temperature ?

6.11 A sample of HI(g) is placed in flask at a pressure of 0.2 atm. At equilibrium the partial
pressure of HI(g) is 0.04 atm. What is Kp for the given equilibrium ?
2HI (g) H2 (g) + I2 (g)
6.12 A mixture of 1.57 mol of N2, 1.92 mol of H2 and 8.13 mol of NH3 is introduced into
a 20 L reaction vessel at 500 K. At this temperature, the equilibrium constant, Kc
for the reaction N2 (g) + 3H2 (g) 2NH3 (g) is 1.7 × 102. Is the reaction mixture at
equilibrium? If not, what is the direction of the net reaction?

6.13 The equilibrium constant expression for a gas reaction is,

NH3  O2 5
4

Kc 
NO4 H2O
6

Write the balanced chemical equation corresponding to this expression.

6.14 One mole of H2O and one mole of CO are taken in 10 L vessel and heated to
725 K. At equilibrium 40% of water (by mass) reacts with CO according to the equation,
H2O (g) + CO (g) H2 (g) + CO2 (g)
Calculate the equilibrium constant for the reaction.

6.15 At 700 K, equilibrium constant for the reaction:


H2 (g) + I2 (g) 2HI (g)
is 54.8. If 0.5 mol L of HI(g) is present at equilibrium at 700 K, what are the
–1

concentration of H2(g) and I2(g) assuming that we initially started with HI(g) and
allowed it to reach equilibrium at 700K?

Rationalised 2023-24

Unit 6.indd 209 9/12/2022 11:58:31 AM


210 chemistry

6.16 What is the equilibrium concentration of each of the substances in the equilibrium
when the initial concentration of ICl was 0.78 M ?
2ICl (g) I2 (g) + Cl2 (g); Kc = 0.14
6.17 Kp = 0.04 atm at 899 K for the equilibrium shown below. What is the equilibrium
concentration of C2H6 when it is placed in a flask at 4.0 atm pressure and allowed
to come to equilibrium?
C2H6 (g) C2H4 (g) + H2 (g)
6.18 Ethyl acetate is formed by the reaction between ethanol and acetic acid and the
equilibrium is represented as:
CH3COOH (l) + C2H5OH (l) CH3COOC2H5 (l) + H2O (l)
(i) Write the concentration ratio (reaction quotient), Qc, for this reaction (note: water
is not in excess and is not a solvent in this reaction)
(ii) At 293 K, if one starts with 1.00 mol of acetic acid and 0.18 mol of ethanol, there
is 0.171 mol of ethyl acetate in the final equilibrium mixture. Calculate the
equilibrium constant.
(iii) Starting with 0.5 mol of ethanol and 1.0 mol of acetic acid and maintaining it at
293 K, 0.214 mol of ethyl acetate is found after sometime. Has equilibrium been
reached?
6.19 A sample of pure PCl 5 was introduced into an evacuated vessel at 473
K. After equilibrium was attained, concentration of PCl 5 was found to be
0.5 × 10–1 mol L–1. If value of Kc is 8.3 × 10–3, what are the concentrations of PCl3 and
Cl2 at equilibrium?
PCl5 (g) PCl3 (g) + Cl2(g)
6.20 One of the reaction that takes place in producing steel from iron ore is the reduction
of iron(II) oxide by carbon monoxide to give iron metal and CO2.
FeO (s) + CO (g) Fe (s) + CO2 (g); Kp = 0.265 atm at 1050K

What are the equilibrium partial pressures of CO and CO2 at 1050 K if the initial
partial pressures are: pCO= 1.4 atm and = 0.80 atm?

6.21 Equilibrium constant, Kc for the reaction


N2 (g) + 3H2 (g) 2NH3 (g) at 500 K is 0.061

At a particular time, the analysis shows that composition of the reaction mixture is
3.0 mol L–1 N2, 2.0 mol L–1 H2 and 0.5 mol L–1 NH3. Is the reaction at equilibrium? If
not in which direction does the reaction tend to proceed to reach equilibrium?

6.22 Bromine monochloride, BrCl decomposes into bromine and chlorine and reaches the
equilibrium:
2BrCl (g) Br2 (g) + Cl2 (g)

for which Kc= 32 at 500 K. If initially pure BrCl is present at a concentration of


3.3 × 10–3 mol L–1, what is its molar concentration in the mixture at equilibrium?

6.23 At 1127 K and 1 atm pressure, a gaseous mixture of CO and CO2 in equilibrium with
soild carbon has 90.55% CO by mass
C (s) + CO2 (g) 2CO (g)

Calculate Kc for this reaction at the above temperature.

Rationalised 2023-24

Unit 6.indd 210 9/12/2022 11:58:31 AM


EQUILIBRIUM 211

6.24 Calculate a) ∆G and b) the equilibrium constant for the formation of NO2 from NO
and O2 at 298K
NO (g) + ½ O2 (g) NO2 (g)
where
∆fG (NO2) = 52.0 kJ/mol
∆fG (NO) = 87.0 kJ/mol
∆fG (O2) = 0 kJ/mol
6.25 Does the number of moles of reaction products increase, decrease or remain same
when each of the following equilibria is subjected to a decrease in pressure by
increasing the volume?
(a) PCl5 (g) PCl3 (g) + Cl2 (g)
(b) CaO (s) + CO2 (g) CaCO3 (s)
(c) 3Fe (s) + 4H2O (g) Fe3O4 (s) + 4H2 (g)
6.26 Which of the following reactions will get affected by increasing the pressure?
Also, mention whether change will cause the reaction to go into forward or
backward direction.
(i) COCl2 (g) CO (g) + Cl2 (g)
(ii) CH4 (g) + 2S2 (g) CS2 (g) + 2H2S (g)
(iii) CO2 (g) + C (s) 2CO (g)
(iv) 2H2 (g) + CO (g) CH3OH (g)
(v) CaCO3 (s) CaO (s) + CO2 (g)
(vi) 4 NH3 (g) + 5O2 (g) 4NO (g) + 6H2O(g)
6.27 The equilibrium constant for the following reaction is 1.6 ×105 at 1024K
H2(g) + Br2(g) 2HBr(g)
Find the equilibrium pressure of all gases if 10.0 bar of HBr is introduced into a
sealed container at 1024K.
6.28 Dihydrogen gas is obtained from natural gas by partial oxidation with steam as per
following endothermic reaction:
CH4 (g) + H2O (g) CO (g) + 3H2 (g)
(a) Write as expression for Kp for the above reaction.
(b) How will the values of Kp and composition of equilibrium mixture be
affected by
(i) increasing the pressure
(ii) increasing the temperature
(iii) using a catalyst?
6.29 Describe the effect of:
a) addition of H2
b) addition of CH3OH
c) removal of CO
d) removal of CH3OH
on the equilibrium of the reaction:
2H2(g) + CO (g) CH3OH (g)
6.30 At 473 K, equilibrium constant Kc for decomposition of phosphorus pentachloride,
PCl5 is 8.3 ×10-3. If decomposition is depicted as,

Rationalised 2023-24

Unit 6.indd 211 9/12/2022 11:58:31 AM


212 chemistry

PCl5 (g) PCl3 (g) + Cl2 (g) ∆rH  = 124.0 kJ mol–1


a) write an expression for Kc for the reaction.
b) what is the value of Kc for the reverse reaction at the same temperature?
c) what would be the effect on Kc if (i) more PCl5 is added (ii) pressure is increased
(iii) the temperature is increased ?
6.31 Dihydrogen gas used in Haber’s process is produced by reacting methane from
natural gas with high temperature steam. The first stage of two stage reaction
involves the formation of CO and H2. In second stage, CO formed in first stage is
reacted with more steam in water gas shift reaction,
CO (g) + H2O (g) CO2 (g) + H2 (g)
If a reaction vessel at 400 °C is charged with an equimolar mixture of CO and steam
such that pco = pH O = 4.0 bar, what will be the partial pressure of H2 at equilibrium?
2
Kp= 10.1 at 400°C
6.32 Predict which of the following reaction will have appreciable concentration of reactants
and products:
a) Cl2 (g) 2Cl (g) Kc = 5 ×10–39
b) Cl2 (g) + 2NO (g) 2NOCl (g) Kc = 3.7 × 108
c) Cl2 (g) + 2NO2 (g) 2NO2Cl (g) Kc = 1.8
6.33 The value of Kc for the reaction 3O2 (g) 2O3 (g) is 2.0 ×10–50 at 25°C. If the
equilibrium concentration of O2 in air at 25°C is 1.6 ×10–2, what is the concentration
of O3?
6.34 The reaction, CO(g) + 3H2(g) CH4(g) + H2O(g)
is at equilibrium at 1300 K in a 1L flask. It also contain 0.30 mol of CO, 0.10 mol of
H2 and 0.02 mol of H2O and an unknown amount of CH4 in the flask. Determine the
concentration of CH4 in the mixture. The equilibrium constant, Kc for the reaction
at the given temperature is 3.90.
6.35 What is meant by the conjugate acid-base pair? Find the conjugate acid/base for
the following species:
HNO2, CN–, HClO4, F –, OH–, CO , and S2–
6.36 Which of the followings are Lewis acids? H2O, BF3, H+, and NH4+
6.37 What will be the conjugate bases for the Brönsted acids: HF, H2SO4 and HCO–3?
6.38 Write the conjugate acids for the following Brönsted bases: NH2–, NH3 and HCOO–.
6.39 The species: H2O, HCO3–, HSO4– and NH3 can act both as Brönsted acids and bases.
For each case give the corresponding conjugate acid and base.
6.40 Classify the following species into Lewis acids and Lewis bases and show how these
act as Lewis acid/base: (a) OH – (b) F – (c) H+ (d) BCl3 .
6.41 The concentration of hydrogen ion in a sample of soft drink is 3.8 × 10–3 M. What is
its pH?
6.42 The pH of a sample of vinegar is 3.76. Calculate the concentration of hydrogen ion
in it.
6.43 The ionization constant of HF, HCOOH and HCN at 298K are 6.8 × 10 –4,
1.8 × 10–4 and 4.8 × 10–9 respectively. Calculate the ionization constants of the
corresponding conjugate base.

Rationalised 2023-24

Unit 6.indd 212 11/30/2022 17:00:00


EQUILIBRIUM 213

6.44 The ionization constant of phenol is 1.0 × 10–10. What is the concentration of phenolate
ion in 0.05 M solution of phenol? What will be its degree of ionization if the solution
is also 0.01M in sodium phenolate?
6.45 The first ionization constant of H2S is 9.1 × 10–8. Calculate the concentration
of HS– ion in its 0.1M solution. How will this concentration be affected if the
solution is 0.1M in HCl also? If the second dissociation constant of H 2S is
1.2 × 10–13, calculate the concentration of S2– under both conditions.
6.46 The ionization constant of acetic acid is 1.74 × 10–5. Calculate the degree of
dissociation of acetic acid in its 0.05 M solution. Calculate the concentration of
acetate ion in the solution and its pH.
6.47 It has been found that the pH of a 0.01M solution of an organic acid is 4.15. Calculate
the concentration of the anion, the ionization constant of the acid and its pKa.
6.48 Assuming complete dissociation, calculate the pH of the following solutions:
(a) 0.003 M HCl (b) 0.005 M NaOH (c) 0.002 M HBr (d) 0.002 M KOH
6.49 Calculate the pH of the following solutions:
a) 2 g of TlOH dissolved in water to give 2 litre of solution.
b) 0.3 g of Ca(OH)2 dissolved in water to give 500 mL of solution.
c) 0.3 g of NaOH dissolved in water to give 200 mL of solution.
d) 1mL of 13.6 M HCl is diluted with water to give 1 litre of solution.
6.50 The degree of ionization of a 0.1M bromoacetic acid solution is 0.132. Calculate the
pH of the solution and the pKa of bromoacetic acid.
6.51 The pH of 0.005M codeine (C18H21NO3) solution is 9.95. Calculate its ionization
constant and pKb.
6.52 What is the pH of 0.001M aniline solution? The ionization constant of aniline
can be taken from Table 6.7. Calculate the degree of ionization of aniline in the
solution. Also calculate the ionization constant of the conjugate acid of aniline.
6.53 Calculate the degree of ionization of 0.05M acetic acid if its pKa value is 4.74.
How is the degree of dissociation affected when its solution also contains
(a) 0.01M (b) 0.1M in HCl ?
6.54 The ionization constant of dimethylamine is 5.4 × 10–4. Calculate its degree of
ionization in its 0.02M solution. What percentage of dimethylamine is ionized if the
solution is also 0.1M in NaOH?
6.55 Calculate the hydrogen ion concentration in the following biological fluids whose
pH are given below:
(a) Human muscle-fluid, 6.83 (b) Human stomach fluid, 1.2
(c) Human blood, 7.38 (d) Human saliva, 6.4.
6.56 The pH of milk, black coffee, tomato juice, lemon juice and egg white are 6.8, 5.0,
4.2, 2.2 and 7.8 respectively. Calculate corresponding hydrogen ion concentration
in each.
6.57 If 0.561 g of KOH is dissolved in water to give 200 mL of solution at 298 K. Calculate
the concentrations of potassium, hydrogen and hydroxyl ions. What is its pH?
6.58 The solubility of Sr(OH)2 at 298 K is 19.23 g/L of solution. Calculate the concentrations
of strontium and hydroxyl ions and the pH of the solution.

Rationalised 2023-24

Unit 6.indd 213 11/30/2022 17:00:00


214 chemistry

6.59 The ionization constant of propanoic acid is 1.32 × 10–5. Calculate the degree of
ionization of the acid in its 0.05M solution and also its pH. What will be its degree
of ionization if the solution is 0.01M in HCl also?
6.60 The pH of 0.1M solution of cyanic acid (HCNO) is 2.34. Calculate the ionization
constant of the acid and its degree of ionization in the solution.
6.61 The ionization constant of nitrous acid is 4.5 × 10–4. Calculate the pH of 0.04 M
sodium nitrite solution and also its degree of hydrolysis.
6.62 A 0.02M solution of pyridinium hydrochloride has pH = 3.44. Calculate the ionization
constant of pyridine.
6.63 Predict if the solutions of the following salts are neutral, acidic or basic:
NaCl, KBr, NaCN, NH4NO3, NaNO2 and KF
6.64 The ionization constant of chloroacetic acid is 1.35 × 10–3. What will be the pH of
0.1M acid and its 0.1M sodium salt solution?
6.65 Ionic product of water at 310 K is 2.7 × 10–14. What is the pH of neutral water at
this temperature?
6.66 Calculate the pH of the resultant mixtures:
a) 10 mL of 0.2M Ca(OH)2 + 25 mL of 0.1M HCl
b) 10 mL of 0.01M H2SO4 + 10 mL of 0.01M Ca(OH)2
c) 10 mL of 0.1M H2SO4 + 10 mL of 0.1M KOH
6.67 Determine the solubilities of silver chromate, barium chromate, ferric hydroxide,
lead chloride and mercurous iodide at 298K from their solubility product constants
given in Table 6.9. Determine also the molarities of individual ions.
6.68 The solubility product constant of Ag 2CrO 4 and AgBr are 1.1 × 10 –12 and
5.0 × 10–13 respectively. Calculate the ratio of the molarities of their saturated
solutions.
6.69 Equal volumes of 0.002 M solutions of sodium iodate and cupric chlorate are
mixed together. Will it lead to precipitation of copper iodate? (For cupric iodate
Ksp = 7.4 × 10–8 ).
6.70 The ionization constant of benzoic acid is 6.46 × 10–5 and Ksp for silver benzoate is
2.5 × 10–13. How many times is silver benzoate more soluble in a buffer of pH 3.19
compared to its solubility in pure water?
6.71 What is the maximum concentration of equimolar solutions of ferrous sulphate and
sodium sulphide so that when mixed in equal volumes, there is no precipitation of
iron sulphide? (For iron sulphide, Ksp = 6.3 × 10–18).
6.72 What is the minimum volume of water required to dissolve 1g of calcium sulphate
at 298 K? (For calcium sulphate, Ksp is 9.1 × 10–6).
6.73 The concentration of sulphide ion in 0.1M HCl solution saturated with hydrogen
sulphide is 1.0 × 10–19 M. If 10 mL of this is added to 5 mL of 0.04 M solution of the
following: FeSO4, MnCl2, ZnCl2 and CdCl2. in which of these solutions precipitation
will take place?

Rationalised 2023-24

Unit 6.indd 214 11/30/2022 17:00:00

You might also like